Tag Archives: IELTS Academic reading

AUSTRALIA’S SPORTING SUCCESS

Reading passage 1

A They play hard, they play often, and they play to win. Australian sports teams win more than their fair share of titles, demolishing rivals with seeming ease. How do they do it? A big part of the secret is an extensive and expensive network of sporting academies underpinned by science and medicine. At the Australian Institute of Sport (AIS), hundreds of youngsters and pros live and train under the eyes of coaches. Another body, the Australian Sports Commission (ASC), finances programmes of excellence in a total of 96 sports for thousands of sportsmen and women. Both provide intensive coaching, training facilities and nutritional advice.

B Inside the academies, science takes centre stage. The AIS employs more than 100 sports scientists and doctors, and collaborates with scores of others in universities and research centres. AIS scientists work across a number of sports, applying skills learned in one – such as building muscle strength in golfers – to others, such as swimming and squash. They are backed up by technicians who design instruments to collect data from athletes. They all focus on one aim: winning. ‘We can’t waste our time looking at ethereal scientific questions that don’t help the coach work with an athlete and improve performance.’ says Peter Fricker, chief of science at AIS.

C A lot of their work comes down to measurement – everything from the exact angle of a swimmers dive to the second-by-second power output of a cyclist. This data is used to wring improvements out of athletes. The focus is on individuals, tweaking performances to squeeze an extra hundredth of a second here, an extra millimetre there. No gain is too slight to bother with. It’s the tiny, gradual improvements that add up to world-beating results. To demonstrate how the system works, Bruce Mason at AIS shows off the prototype of a 3D analysis tool for studying swimmers. A wire-frame model of a champion swimmer slices through the water, her arms moving in slow motion. Looking side-on, Mason measures the distance between strokes. From above, he analyses how her spine swivels. When fully developed, this system will enable him to build a biomechanical profile for coaches to use to help budding swimmers. Mason’s contribution to sport also includes the development of the SWAN (SWimming ANalysis) system now used in Australian national competitions. It collects images from digital cameras running at 50 frames a second and breaks down each part of a swimmers performance into factors that can be analysed individually – stroke length, stroke frequency, average duration of each stroke, velocity, start, lap and finish times, and so on. At the end of each race, SWAN spits out data on each swimmer.

D ‘Take a look.’ says Mason, pulling out a sheet of data. He points out the data on the swimmers in second and third place, which shows that the one who finished third actually swam faster. So why did he finish 35 hundredths of a second down? ‘His turn times were 44 hundredths of a second behind the other guy,’ says Mason. ‘If he can improve on his turns, he can do much better.’ This is the kind of accuracy that AIS scientists’ research is bringing to a range of sports. With the Cooperative Research Centre for Micro Technology in Melbourne, they are developing unobtrusive sensors that will be embedded in an athlete’s clothes or running shoes to monitor heart rate, sweating, heat production or any other factor that might have an impact on an athlete’s ability to run. There’s more to it than simply measuring performance. Fricker gives the example of athletes who may be down with coughs and colds 11 or 12 times a year. After years of experimentation, AIS and the University of Newcastle in New South Wales developed a test that measures how much of the immune-system protein immunoglobulin A is present in athletes’ saliva. If IgA levels suddenly fall below a certain level, training is eased or dropped altogether. Soon, IgA levels start rising again, and the danger passes. Since the tests were introduced, AIS athletes in all sports have been remarkably successful at staying healthy.

E Using data is a complex business. Well before a championship, sports scientists and coaches start to prepare the athlete by developing a ‘competition model’, based on what they expect will be the winning times. ‘You design the model to make that time.’ says Mason. ‘A start of this much, each free-swimming period has to be this fast, with a certain stroke frequency and stroke length, with turns done in these times’. All the training is then geared towards making the athlete hit those targets, both overall and for each segment of the race. Techniques like these have transformed Australia into arguably the world’s most successful sporting nation.

F Of course, there’s nothing to stop other countries copying – and many have tried. Some years ago, the AIS unveiled coolant-lined jackets for endurance athletes. At the Atlanta Olympic Games in 1996, these sliced as much as two per cent off cyclists’ and rowers times. Now everyone uses them. The same has happened to the altitude tent’, developed by AIS to replicate the effect of altitude training at sea level. But Australia’s success story is about more than easily copied technological fixes, and up to now no nation has replicated its all-encompassing system.

Questions 1-7

Reading Passage 1 has six sections, A-F. Which paragraph contains the following information? Write the correct letter A-F in boxes 1-7 on your answer sheet. NB You may use any letter more than once

  1. a reference to the exchange of expertise between different sports
  2. an explanation of how visual imaging is employed in investigations
  3. a reason for narrowing the scope of research activity
  4. how some AIS ideas have been reproduced
  5. how obstacles to optimum achievement can be investigated
  6. an overview of the funded support of athletes
  7. how performance requirements are calculated before an event


    Questions 8-11. Classify the following techniques according to whether the writer states they

    A are currently exclusively used by Australians
    B will be used in the future by Australians
    C are currently used by both Australians and their rivals

    Write the correct letter A, B, or C in boxes 8-11 on your answer sheet.
  8. cameras
  9. sensors
  10. protein tests
  11. altitude tents

    Questions 12 and 13. Choose NO MORE THAN THREE WORDS AND/OR A NUMBER from the Reading Passage 1 for each answer. Write your answers in boxes 12 and 13 on your answer sheet.
  12. What is produced to help an athlete plan their performance in an event?
  13. By how much did some cyclists’ performance improve at the 1996 Olympic Games?

    Reading passage 2

    DELIVERING THE GOODS

    A International trade is growing at a startling pace. While the global economy has been expanding at a bit over 3% a year, the volume of trade has been rising at a compound annual rate of about twice that. Foreign products, from meat to machinery, play a more important role in almost every economy in the world, and foreign markets now tempt businesses that never much worried about sales beyond their nation’s borders.

    B What lies behind this explosion in international commerce? The general worldwide decline in trade barriers, such as customs duties and import quotas, is surely one explanation. The economic opening of countries that have traditionally been minor players is another. But one force behind the import-export boom has passed all but unnoticed: the rapidly falling cost of getting goods to market. Theoretically, in the world of trade, shipping costs do not matter. Goods, once they have been made, are assumed to move instantly and at no cost from place to place. The real world, however, is full of frictions. Cheap labour may make Chinese clothing competitive in America, but if delays in shipment tie up working capital and cause winter coats to arrive in spring, trade may lose its advantages.

    C At the turn of the 20th century, agriculture and manufacturing were the two most important sectors almost everywhere, accounting for about 70% of total output in Germany, Italy and France, and 40-50% in America, Britain and Japan. International commerce was therefore dominated by raw materials, such as wheat, wood and iron ore, or processed commodities, such as meat and steel. But these sorts of products are heavy and bulky and the cost of transporting them relatively high.

    D Countries still trade disproportionately with their geographic neighbours. Over time, however, world output has shifted into goods whose worth is unrelated to their size and weight. Today, it is finished manufactured products that dominate the flow of trade, and, thanks to technological advances such as lightweight components, manufactured goods themselves have tended to become lighter and less bulky. As a result, less transportation isrequired for every dollar’s worth of imports or exports.

    E To see how this influences trade, consider the business of making disk drives for computers. Most ofthe world’s disk-drive manufacturing is concentrated in South-east Asia. This is possible only because disk drives, while valuable, are small and light and so cost little to ship. Computer manufacturers in Japan or Texas will not face hugely bigger freight bills if they import drives from Singapore rather than purchasing them on the domestic market. Distance therefore poses no obstacle to the globalisation of the disk-drive industry.

    F This is even more true of the fast-growing information industries. Films and compact discs cost little to transport, even by aeroplane. Computer software can be ‘exported’ without ever loading it onto a ship, simply by transmitting it over telephone lines from one country to another, so freight rates and cargo-handling schedules become insignificant factors in deciding where to make the product. Businesses can locate based on other considerations, such as the availability of labour, while worrying less about the cost of delivering their output.

    G In many countries deregulation has helped to drive the process along. But, behind the scenes, a series of technological innovations known broadly as containerisation and inter-modal transportation has led to swift productivity improvements in cargo-handling. Forty years ago, the process of exporting or importing involved a great many stages of handling, which risked portions of the shipment being damaged or stolen along the way. The invention of the container crane made it possible to load and unload containers without capsizing the ship and the adoption of standard container sizes allowed almost any box to be transported on any ship. By 1967, dual-purpose ships, carrying loose cargo in the hold and containers on the deck, were giving way to all-container vessels that moved thousands of boxes at a time.

    H The shipping container transformed ocean shipping into a highly efficient, intensely competitive business. But getting the cargo to and from the dock was a different story. National governments, by and large, kept a much firmer hand on truck and railroad tariffs than on charges for ocean freight. This started changing, however, in the mid-1970s, when America began to deregulate its transportation industry. First airlines, then road hauliers and railways, were freed from restrictions on what they could carry, where they could haul it and what price they could charge. Big productivity gains resulted. Between 1985 and 1996, for example, America’s freight railways dramatically reduced their employment, trackage, and their fleets of locomotives – while increasing the amount of cargo they hauled. Europe’s railways have also shown marked, albeit smaller, productivity improvements.

    I In America the period of huge productivity gains in transportation may be almost over, but in most countries the process still has far to go. State ownership of railways and airlines, regulation of freight rates and toleration of anti- competitive practices, such as cargo-handling monopolies, all keep the cost of shipping unnecessarily high anddeter international trade. Bringing these barriers down would help the world’s economies grow even closer.

    Questions 14-17. Reading Passage 2 has six sections, A-I.Which paragraph contains the following information?Write the correct letter A-I in boxes 14-17 on your answer sheet.
  14. a suggestion for improving trade in the future
  15. the effects of the introduction of electronic delivery
  16. the similar cost involved in transporting a product from abroad or from a local supplier
  17. the weakening relationship between the value of goods and the cost of their delivery

    Questions 18-22. Do the following statements agree with the information given in Reading Passage 2? In boxes 18-22 on your answer sheet, write
    TRUE if the statement agrees with the information
    FALSE if the statement contradicts the information
    NOT GIVEN if there is no information on this
  18. International trade is increasing at a greater rate than the world economy.
  19. Cheap labour guarantees effective trade conditions.
  20. Japan imports more meat and steel than France.
  21. Most countries continue to prefer to trade with nearby nations.
  22. Small computer components are manufactured in Germany.
Questions 23-26

Complete the summary using the list of words, A-K, below. Write the correct letter, A-K, in boxes 23-26 on your answer sheet.

THE TRANSPORT REVOLUTION

Modern cargo-handling methods have had a significant effect on (23) ……………….. as the business of moving freight around the world becomes increasingly streamlined. Manufacturers of computers, for instance, are able to import

(24) ……………….. from overseas, rather than having to rely on a local supplier. The introduction of (25)………………….. has meant that bulk cargo can be safely and efficiently moved over long distances. While international shipping is now efficient, there is still a need for governments to reduce (26) ………………..in order to free up the domestic cargo sector.

A. tariffsB. componentsC. container shipsD. output
E. employeesF. insurance costsG. tradeH. freight
I. faresJ. softwareK. international standards

Reading passage 3.

Climate change and the Inuit


A
Unusual incidents are being reported across the Arctic. Inuit families going off on snowmobiles to prepare their summer hunting camps have found themselves cut off from home by a sea of mud,following early thaws. There are reports of igloos losing their insulating properties as the snow drips and refreezes, of lakes draining into the sea as permafrost melts, and sea ice breaking up earlier than usual, carrying seals beyond the reach of hunters. Climate change may still be a rather abstract idea to most of us, but in the Arctic it is already having dramatic effects – if summertime ice continues to shrink at its present rate, the Arctic Ocean could soon become virtually ice-free in summer. The knock-on effects are likely to include more warming, cloudier skies, increased precipitation and higher sea levels. Scientists are increasingly keen to find out what’s going on because they consider the Arctic the ‘canary in the mine’ for global warming – a warning of what’s in store for the rest of the world.


B For the Inuit the problem is urgent. They live in precarious balance with one of the toughest environments on earth. Climate change, whatever its causes, is a direct threat to their way of life. Nobody knows the Arctic as well as the locals, which is why they are not content simply to stand back and let outside experts tell them what’s happening. In Canada, where the Inuit people are jealously guarding their hard-won autonomy in the country’s newest territory, Nunavut, they believe their best hope of survival in this changing environment lies in combining their ancestral knowledge with the best of modern science. This is a challenge in itself.

C The Canadian Arctic is a vast, treeless polar desert that’s covered with snow for most of the year. Venture into this terrain and you get some idea of the hardships facing anyone who calls this home. Farming is out of the question and nature offers meagre pickings. Humans first settled in the Arctic a mere 4,500 years ago, surviving by exploiting sea mammals and fish. The environment tested them to the limits: sometimes the colonists were successful, sometimes they failed and vanished. But around a thousand years ago, one group emerged that was uniquely well adapted to cope with the Arctic environment. These Thule people moved in from Alaska, bringing kayaks, sleds, dogs, pottery and iron tools. They are the ancestors of today’s Inuit people.

D Life for the descendants of the Thule people is still harsh. Nunavut is 1.9 million square kilometres of rock and ice, and a handful of islands around the North Pole. It’s currently home to 2,500 people, all but a handful of them indigenous Inuit. Over the past 40 years, most have abandoned their nomadic ways and settled in the territory’s 28 isolated communities, but they still rely heavily on nature to provide food and clothing.Provisions available in local shops have to be flown into Nunavut on one of the most costly air networks in the world, or brought by supply ship during the few ice-free weeks of summer. It would cost a family around £7,000 a year to replace meat they obtained themselves through hunting with imported meat. Economic opportunities are scarce, and for many people state benefits are their only income.

E While the Inuit may not actually starve if hunting and trapping are curtailed by climate change, there has certainly been an impact on people’s health. Obesity, heart disease and diabetes are beginning to appear in a people for whom these have never before been problems. There has been a crisis of identity as the traditional skills of hunting, trapping and preparing skins have begun to disappear. In Nunavut’s ‘igloo and email’ society, where adults who were born in igloos have children who may never have been out on the land, there’s a high incidence of depression.

F With so much at stake, the Inuit are determined to play a key role in teasing out the mysteries of climate change in the Arctic. Having survived there for centuries, they believe their wealth of traditional knowledge is vital to the task. And Western scientists are starting to draw on this wisdom, increasingly referred to as ‘Inuit Qaujimajatugangit’, or IQ. ‘In the early days scientists ignored us when they came up here to study anything. They just figured these people don’t know very much so we won’t ask them,’ says John Amagoalik, an Inuit leader and politician. ‘But in recent years IQ has had much more credibility and weight.’ In fact it is now a requirement for anyone hoping to get permission to do research that they consult the communities, who are helping to set the research agenda to reflect their most important concerns. They can turn down applications from scientists they believe will work against their interests, or research projects that will impinge too much on their daily lives and traditional activities.

G Some scientists doubt the value of traditional knowledge because the occupation of the Arctic doesn’t go back far enough. Others, however, point out that the first weather stations in the far north date back just 50 years.There are still huge gaps in our environmental knowledge, and despite the scientific onslaught, many predictions are no more than best guesses. IQ could help to bridge the gap and resolve the tremendous uncertainty about how much of what we’re seeing is natural capriciousness and how much is the consequence of human activity.

Questions 27-32. Choose the correct heading for paragraphs B-G from the list of headings below.

List of Headings

i The reaction of the Inuit community to climate change

ii Understanding of climate change remains limited

iii Alternative sources of essential supplies

iv Respect for Inuit opinion grows

v A healthier choice of food

vi A difficult landscape

vii Negative effects on well-being

viii Alarm caused by unprecedented events in the Arctic

ix The benefits of an easier existence

27 Paragraph B

28 Paragraph C

29 Paragraph D

30 Paragraph E

31 Paragraph F

32 Paragraph G

Questions 33-40. Complete the summary of paragraphs C and D below. Choose NO MORE THAN TWO WORDS from paragraphs C and D for each answer.

If you visit the Canadian Arctic, you immediately appreciate the problems faced by people for whom this is home. It would clearly be impossible for the people to engage in (33)……………….. as a means of supporting themselves.

For thousands of years they have had to rely on catching (34) ……………….. and (35) ……………….. as a means of sustenance. The harsh surroundings saw many who tried to settle there pushed to their limits, although some were successful. The (36)………………..people were an example of the latter and for them the environment did not prove unmanageable. For the present inhabitants, life continues to be a struggle. The territory of Nunavut consists of little more than ice, rock and a few (37)……………….. In recent years, many of them have been obliged to give up their (38) ..………….. lifestyle, but they continue to depend mainly on (39)their food and clothes. (40)……………….. produce is particularly expensive.


How did it go? Please share your feedback in the comment section below:
Do you need a copy of this test? Download PDF

Show answer
AUSTRALIA’S SPORTING SUCCESS
  1. B
  2. C
  3. B
  4. F
  5. D
  6. A
  7. E
  8. A
  9. B
  10. A
  11. C
  12. (a) competition model
  13. (by) 2%
  14. I
  15. F
  16. E
  17. D
  18. TRUE
  19. FALSE
  20. NOT GIVEN
  21. TRUE
  22. NOT GIVEN
  23. G
  24. B
  25. C
  26. A
  27. i
  28. vi
  29. iii
  30. vii
  31. iv
  32. ii
  33. farming
  34. sea mammals, fish IN EITHER ORDER
  35. sea mammals, fish IN EITHER ORDER
  36. Thule
  37. islands
  38. nomadic
  39. nature
  40. imported

Attitudes to Language

Reading passage 1

It is not easy to be systematic and objective about language study. Popular linguistic debate regularly deteriorates into invective and polemic. Language belongs to everyone, so most people feel they have a right to hold an opinion about it. And when opinions differ, emotions can run high. Arguments can start as easily over minor points of usage as over major policies of linguistic education.

Language, moreover, is a very public behaviour, so it is easy for different usages to be noted and criticised. No part of society or social behaviour is exempt: linguistic factors influence how we judge personality, intelligence, social status, educational standards, job aptitude, and many other areas of identity and social survival. As a result, it is easy to hurt, and to be hurt, when language use is unfeelingly attacked.

In its most general sense, prescriptivism is the view that one variety of language has an inherently higher value than others, and that this ought to be imposed on the whole of the speech community. The view is propounded especially in relation to grammar and vocabulary, and frequently with reference to pronunciation. The variety which is favoured, in this account, is usually a version of the ‘standard’ written language, especially as encountered in literature, or in the formal spoken language which most closely reflects this style. Adherents to this variety are said to speak or write ‘correctly’; deviations from it are said to be ‘incorrect!

All the main languages have been studied prescriptively, especially in the 18th-century approach to the writing of grammars and dictionaries. The aims of these early grammarians were threefold: (a) they wanted to codify the principles of their languages, to show that there was a system beneath the apparent chaos of usage, (b) they wanted a means of settling disputes over usage, and (c) they wanted to point out what they felt to be common errors, in order to ‘improve’ the language. The authoritarian nature of the approach is best characterized by its reliance on ‘rules’ of grammar. Some usages are ‘prescribed,’ to be learned and followed accurately; others are ‘proscribed,’ to be avoided. In this early period, there were no half- measures: usage was either right or wrong, and it was the task of the grammarian not simply to record alternatives but to pronounce judgment upon them

These attitudes are still with us, and they motivate a widespread concern that linguistic standards should be maintained. Nevertheless, there is an alternative point of view that is concerned less with standards than with the facts of linguistic usage. This approach is summarised in the statement that it is the task of the grammarian to describe, not prescribe to record the facts of linguistic diversity, and not to attempt the impossible tasks of evaluating language variation or halting language change. In the second half of the 18th century, we already find advocates of this view, such as Joseph Priestley, whose Rudiments of English Grammar (1761) insists that ‘the custom of speaking is the original and only just standard of any language! Linguistic issues, it is argued, cannot be solved by logic and legislation. And this view has become the tenet of the modern linguistic approach to grammatical analysis.

In our own time, the opposition between ‘descriptivists’ and ‘prescriptivists’ has often become extreme, with both sides painting unreal pictures of the other. Descriptive grammarians have been presented as people who do not care about standards, because of the way they see all forms of usage as equally valid. Prescriptive grammarians have been presented as blind adherents to a historical tradition. The opposition has even been presented in quasi- political terms – of radical liberalism vs elitist conservatism.

Questions 1-8. Do the following statements agree with the claims of the writer in Reading Passage 1? In boxes 1-8 in your answer sheet, write:

YES if the statement agrees with the claims of the writer
NO if the statement contradicts the claims of the writer
NOT GIVEN if it is impossible to say what the writer thinks about this

  1. There are understandable reasons why arguments occur about language.
  2. People feel more strongly about language education than about small differences in language usage.
  3. Our assessment of a person’s intelligence is affected by the way he or she uses language.
  4. Prescriptive grammar books cost a lot of money to buy in the 18th century.
  5. Prescriptivism still exists today.
  6. According to descriptivists it is pointless to try to stop language change.
  7. Descriptivism only appeared after the 18th century.
  8. Both descriptivists and prescriptivists have been misrepresented.

Questions 9-12. Complete the summary using the list of words, A-l, below.

The language debate

According to (9)…………………… there is only one correct form of language. Linguists who take this approach to language place great importance on grammatical (10) ……………………. Conversely, the view of (11) ………….., such as Joseph Priestley, is that grammar should be based on (12) ………………….

A descriptivists

B language expert

C popular speech

D formal language

E evaluation

F rules

G modern linguists

H prescriptivists

I change 

Question 13. Choose the correct letter A. B, C or D.

What is the writer’s purpose in Reading Passage?

A to argue in favour of a particular approach to writing dictionaries and grammar books

B to present a historical account of differing views of language

C to describe the differences between spoken and written language

D to show how a certain view of language has been discredited

Tidal Power

A Operating on the same principle as wind turbines, the power in sea turbines comes from tidal currents which turn blades similar to ships’ propellers, but, unlike wind, the tides are predictable and the power input is constant. The technology raises the prospect of Britain becoming self-sufficient in renewable energy and drastically reducing its carbon dioxide emissions. If tide, wind and wave power are all developed, Britain would be able to close gas, coal and nuclear power plants and export renewable power to other parts of Europe. Unlike wind power, which Britain originally developed and then abandoned for 20 years allowing the Dutch to make it a major industry, undersea turbines could become a big export earner to island nations such as Japan and New Zealand.

B Tidal sites have already been identified that will produce one sixth or more of the UK’s power – and at prices competitive with modern gas turbines and undercutting those of the already ailing nuclear industry. One site alone, the Pentland Firth, between Orkney and mainland Scotland, could produce 10% of the country’s electricity with banks of turbines under the sea, and another at Alderney in the Channel Islands three times the 1,200 megawatts of Britain’s largest and newest nuclear plant, Sizewell B, in Suffolk. Other sites identified include the Bristol Channel and the west coast of Scotland, particularly the channel between Campbeltown and Northern Ireland.

C Work on designs for the new turbine blades and sites are well advanced at the University of Southampton’s sustainable energy research group. The first station is expected to be installed off Lynmouth in Devon shortly to test the technology in a venture jointly funded by the department of Trade and Industry and the European Union. AbuBakr Bahaj, in charge of the Southampton research, said: The prospects for energy from tidal currents are far better than from wind because the flows of water are predictable and constant. The technology for dealing with the hostile saline environment under the sea has been developed in the North Sea oil industry and much is already known about turbine blade design, because of wind power and ship propellers. There are a few technical difficulties, but I believe in the next five to ten years we will be installing commercial marine turbine farms.’ Southampton has been awarded £215,000 over three years to develop the turbines and is working with Marine Current Turbines, a subsidiary of IT power, on the Lynmouth project. EU research has now identified 106 potential sites for tidal power, 80% round the coasts of Britain. The best sites are between islands or around heavily indented coasts where there are strong tidal currents.

D A marine turbine blade needs to be only one third of the size of a wind generator to produce three times as much power. The blades will be about 20 metres in diameter, so around 30 metres of water is required. Unlike wind power, there are unlikely to be environmental objections. Fish and other creatures are thought unlikely to be at risk from the relatively slow-turning blades. Each turbine will be mounted on a tower which will connect to the national power supply grid via underwater cables. The towers will stick out of the water and be lit, to warn shipping, and also be designed to be lifted out of the water for maintenance and to clean seaweed from the blades.

E Dr Bahaj has done most work on the Alderney site, where there are powerful currents. The single undersea turbine farm would produce far more power than needed for the Channel Islands and most would be fed into the French Grid and be re-imported into Britain via the cable under the Channel.

F One technical difficulty is cavitation, where low pressure behind a turning blade causes air bubbles. These can cause vibration and damage the blades of the turbines. Dr Bahaj said: ‘We have to test a number of blade types to avoid this happening or at least make sure it does not damage the turbines or reduce performance. Another slight concern is submerged debris floating into the blades. So far we do not know how much of a problem it might be. We will have to make the turbines robust because the sea is a hostile environment, but all the signs that we can do it are good.’

Questions 14-17

Reading Passage 2 has six paragraphs, A-F. Which paragraph contains the following information? NB You may use any letter more than once.

14 the location of the first test site
15 a way of bringing the power produced on one site back into Britain
16 a reference to a previous attempt by Britain to find an alternative source of energy
17 mention of the possibility of applying technology from another industry

Questions 18-22. Choose FIVE Letters A-J

Which FIVE of the following claims about tidal power are made by the writer?

A It is a more reliable source of energy than wind power.

B It would replace all other forms of energy in Britain.

C Its introduction has come as a result of public pressure.

D It would cut down on air pollution.

E It could contribute to the closure of many existing power stations ln Britain.

F It could be a means of increasing national income.

G It could face a lot of resistance from other fuel industries.

H It could be sold more cheaply than any other type of fuel.

I It could compensate for the shortage of inland sites for energy production. J It is best produced in the vicinity of coastlines with particular features.

Questions 23-26

Label the diagram below. Choose NO MORE THAN TWO WORDS from the passage for each answer. An Undersea Turbine

Information Theory – The Big Idea

A In April 2002 an event took place which demonstrated one of the many applications of information theory. The space probe, Voyager I, launched in 1977, had sent back spectacular images of Jupiter and Saturn and then soared out of the Solar System on a one-way mission to the stars. After 25 years of exposure to the freezing temperatures of deep space, the probe was beginning to show its age. Sensors and circuits were on the brink of failing and NASA experts realised that they had to do something or lose contact with their probe forever. The solution was to get a message to Voyager I to instruct it to use spares to change the failing parts. With the probe 12 billion kilometres from Earth, this was not an easy task. By means of a radio dish belonging to NASA’s Deep Space Network, the message was sent out into the depths of space. Even travelling at the speed of light, it took over 11 hours to reach its target, far beyond the orbit of Pluto. Yet, incredibly, the little probe managed to hear the faint call from its home planet, and successfully made the switchover.

B It was the longest-distance repair job in history, and a triumph for the NASA engineers. But it also highlighted the astonishing power of the techniques developed by American communications engineer Claude Shannon, who had died just a year earlier. Born in 1916 in Petoskey, Michigan, Shannon showed an early talent for maths and for building gadgets, and made breakthroughs in the foundations of computer technology when still a student. While at Bell Laboratories, Shannon developed information theory, but shunned the resulting acclaim. In the 1940s, he single-handedly created an entire science of communication which has since inveigled its way into a host of applications, from DVDs to satellite communications to bar codes – any area, in short, where data has to be conveyed rapidly yet accurately.

C This all seems light years away from the down-to-earth uses Shannon originally had for his work, which began when he was a 22-year-old graduate engineering student at the prestigious Massachusetts Institute of Technology in 1939. He set out with an apparently simple aim: to pin down the precise meaning of the concept of ‘information’. The most basic form of information, Shannon argued, is whether something is true or false – which can be captured in the binary unit, or ‘bit’, of the form 1 or 0. Having identified this fundamental unit, Shannon set about defining otherwise vague ideas about information and how to transmit it from place to place. In the process he discovered something surprising: it is always possible to guarantee information will get through random interference – ‘noise’ – intact.

D Noise usually means unwanted sounds which interfere with genuine information. Information theory generalises this idea via theorems that capture the effects of noise with mathematical precision. In particular, Shannon showed that noise sets a limit on the rate at which information can pass along communication channels while remaining error-free. This rate depends on the relative strengths of the signal and noise travelling down the communication channel, and on its capacity (its ‘bandwidth’). The resulting limit, given in units of bits per second, is the absolute maximum rate of error-free communication given signal strength and noise level. The trick, Shannon showed, is to find ways of packaging up – ‘coding’ – information to cope with the ravages of noise, while staying within the information-carrying capacity – ‘bandwidth’ – of the communication system being used.

E Over the years scientists have devised many such coding methods, and they have proved crucial in many technological feats. The Voyager spacecraft transmitted data using codes which added one extra bit for every single bit of information; the result was an error rate of just one bit in 10,000 – and stunningly clear pictures of the planets. Other codes have become part of everyday life – such as the Universal Product Code, or bar code, which uses a simple error-detecting system that ensures supermarket check-out lasers can read the price even on, say, a crumpled bag of crisps. As recently as 1993, engineers made a major breakthrough by discovering so-called turbo codes – which come very close to Shannon’s ultimate limit for the maximum rate that data can be transmitted reliably, and now play a key role in the mobile videophone revolution.

F Shannon also laid the foundations of more efficient ways of storing information, by stripping out superfluous (‘redundant’) bits from data which contributed little real information. As mobile phone text messages like ‘I CN C U’ show, it is often possible to leave out a lot of data without losing much meaning. As with error correction, however, there’s a limit beyond which messages become too ambiguous. Shannon showed how to calculate this limit, opening the way to the design of compression methods that cram maximum information into the minimum space.

Questions 27-32

Reading Passage 3 has six paragraphs, A-F. Which paragraph contains the following information?

27 an explanation of the factors affecting the transmission of information
28 an example of how unnecessary information can be omitted
29 a reference to Shannon`s attitude to fame
30 details of a machine capable of interpreting incomplete information
31 a detailed account of an incident involving information theory
32 a reference to what Shannon initially intended to achieve in his research

Questions 33-37

Complete the notes below. Choose NO MORE THAN TWO WORDS from the passage for each answer

The Voyager l Space Probe

The probe transmitted pictures of both (33) ……………….,and ……………. , then left the (34) ……………. The freezing temperatures were found to have a negative effect on parts of the space probe. Scientists feared that both the (35)……………….. and …………………….. were about to stop working. The only hope was to tell the probe to replace them with (36)…………..… but distance made communication with the probe difficult. A (37)…………….. was used to transmit the message at the speed of light. The message was picked up by the probe and the switchover took place.

Questions 38-40

Do the following statements agree with the information given in Reading Passage 3? In boxes 38-40 on your answer sheet write

TRUE if the statement agrees with the information
FALSE if the statement contradicts the information
NOT GIVEN if there is no information on this

38. The concept of describing something as true or false was the starting point for Shannon in his attempts to send messages over distances.

39. The amount of information that can be sent in a given time period is determined with reference to the signal strength and noise level.

40. Products have now been developed which can convey more information than Shannon had anticipated as possible.

How did it go? Please share your feedback in the comment section below:
Do you need a copy of this test? Download PDF

Show answer
Attitudes to Language
  1. Yes
  2. No
  3. Yes
  4. Not given
  5. Yes
  6. Yes
  7. No
  8. Yes
  9. H
  10. F
  11. A
  12. C
  13. B
  14. C
  15. E
  16. A
  17. C
  18. A, D, E, F, J IN ANY ORDER
  19. A, D, E, F, J IN ANY ORDER
  20. A, D, E, F, J IN ANY ORDER
  21. A, D, E, F, J IN ANY ORDER
  22. A, D, E, F, J IN ANY ORDER
  23. Maintenance
  24. Slow (turning)
  25. Low pressure
  26. Cavitation
  27. D
  28. F
  29. B
  30. E
  31. A
  32. C
  33. Jupiter and Saturn (in any order) Both are required for one mark
  34. Solar system
  35. Sensors and circuits (in any order) Both are required for one mark
  36. Spares
  37. Radio dish
  38. True
  39. True
  40. False

Aphantasia: A life without mental images 

Reading Passage 1

You should spend about 20 minutes on Questions 1–13, which are based on Reading Passage 1 below.

Close your eyes and imagine walking along a sandy beach and then gazing over the horizon as the Sun rises. How clear is the image that springs to mind?

Most people can readily conjure images inside their head – known as their mind’s eye. But this year scientists have described a condition, Aphantasia, in which some people are unable to visualise mental images. Niel Kenmuir, from Lancaster, has always had a blind mind’s eye. He knew he was different even in childhood. “My stepfather, when I couldn’t sleep, told me to count sheep, and he explained what he meant, I tried to do it and I couldn’t,” he says. “I couldn’t see any sheep jumping over fences, there was nothing to count.”

Our memories are often tied up in images, think back to a wedding or first day at school. As a result, Niel admits, some aspects of his memory are “terrible”, but he is very good at remembering facts. And, like others with Aphantasia, he struggles to recognise faces. Yet he does not see Aphantasia as a disability, but simply a different way of experiencing life.

Mind’s eye blind

Ironically, Niel now works in a bookshop, although he largely sticks to the non-fiction aisles. His condition begs the question of what is going on inside his picture-less mind. I asked him what happens when he tries to picture his fiancee. “This is the hardest thing to describe, what happens in my head when I think about things,” he says. “When I think about my fiancee there is no image, but I am definitely thinking about her, I know today she has her hair up at the back, she’s brunette. But I’m not describing an image I am looking at, I’m remembering features about her, that’s the strangest thing and maybe that is a source of some regret.”

The response from his mates is very sympathetic: “You’re weird.” But while Niel is very relaxed about his inability to picture things, it is often a cause of distress for others. One person who took part in a study into Aphantasia said he had started to feel “isolated” and “alone” after discovering that other people could see images in their heads. Being unable to reminisce about his mother years after her death led to him being “extremely distraught”. 

The super-visualiser 

At the other end of the spectrum is children’s book illustrator, Lauren Beard, whose work on the Fairytale Hairdresser series will be familiar to many six-year-olds. Her career relies on the vivid images that leap into her mind’s eye when she reads text from her author. When I met her in her box-room studio in Manchester, she was working on a dramatic scene in the next book. The text describes a baby perilously climbing onto a chandelier. 

“Straightaway I can visualise this grand glass chandelier in some sort of French kind of ballroom, and the little baby just swinging off it and really heavy thick curtains,” she says. “I think I have a strong imagination, so I can create the world and then keep adding to it so it gets sort of bigger and bigger in my mind and the characters too they sort of evolving. I couldn’t really imagine what it’s like to not imagine, I think it must be a bit of a shame really.” 

Not many people have mental imagery as vibrant as Lauren or as blank as Niel. They are the two extremes of visualisation. Adam Zeman, a professor of cognitive and behavioural neurology, wants to compare the lives and experiences of people with Aphantasia and its polar-opposite Hyperphantasia. His team, based at the University of Exeter, coined the term Aphantasia this year in a study in the journal Cortex. 

Prof Zeman tells the BBC: “People who have contacted us say they are really delighted that this has been recognised and has been given a name because they have been trying to explain to people for years that there is this oddity that they find hard to convey to others.” How we imagine is clearly very subjective – one person’s vivid scene could be another’s a grainy picture. But Prof Zeman is certain that Aphantasia is real. People often report being able to dream in pictures, and there have been reported cases of people losing the ability to think in images after a brain injury. 

He is adamant that Aphantasia is “not a disorder” and says it may affect up to one in 50 people. But he adds: “I think it makes quite an important difference to their experience of life because many of us spend our lives with imagery hovering somewhere in the mind’s eye which we inspect from time to time, it’s the variability of human experience.”Top of Form

Questions 1–8. Do the following statements agree with the information in the IELTS reading text? In boxes 1- 8 on your answer sheet, write 

TRUE, if the statement agrees with the information 
FALSE, if the statement contradicts the information 
NOT GIVEN, if there is no information on this 

1.Aphantasia is a condition, which describes people, for whom it is hard to visualise mentalimages.
2.Niel Kenmuir was unable to count sheep in his head.
3.People with Aphantasia struggle to remember personal traits and clothes of differentpeople.
4.Niel regrets that he cannot portray an image of his fiancee in his mind.
5.Inability to picture things in someone’s head is often a cause of distress for a person.
6.All people with Aphantasia start to feel ‘isolated’ or ‘alone’ at some point in their lives.
7.Lauren Beard’s career depends on her imagination.
8.The author met Lauren Beard when she was working on a comedy scene in her next book.

Questions 9–13 . Complete the sentences below. Write NO MORE THAN TWO WORDS from the passage for each answer. Write your answers in boxes 9-13 on your answer sheet. 

9. Only a small fraction of people have imagination as_________ as Lauren does.
10. Hyperphantasia is __________ to Aphantasia.
11. There are lots of subjectivity in comparing people’s imagination – somebody’s vivid scenecould be another person’s __________.
12. Prof Zeman is _________ that Aphantasia is not an illness.
13. Many people spend their lives with _________ somewhere in the mind’s eye.

Reading Passage 2 

You should spend about 20 minutes on Questions 14–26, which are based on Reading Passage 2 below.

Life lessons from villains, crooks and gangsters 

A. A notorious Mexican drug baron’s audacious escape from prison in July doesn’t, atfirst, appear to have much to teach corporate boards. But some in the business world suggest otherwise. Beyond the morally reprehensible side of criminals’ work, some business gurus say organised crime syndicates, computer hackers, pirates and others operating outside the law could teach legitimate corporations a thing or two about how to hustle and respond to rapid change. 

B. Far from encouraging illegality, these gurus argue that – in the same way, bigcorporations sometimes emulate start-ups – business leaders could learn from the underworld about flexibility, innovation and the ability to pivot quickly. “There is a nimbleness to criminal organisations that legacy corporations [with large, complex layers of management] don’t have,” said Marc Goodman, head of the Future Crimes Institute and global cyber-crime advisor. While traditional businesses focus on rules they have to follow, criminals look to circumvent them. “For criminals, the sky is the limit and that creates the opportunity to think much, much bigger.” 

C. Joaquin Guzman, the head of the Mexican Sinaloa drug cartel, for instance, slipped outof his prison cell through a tiny hole in his shower that led to a mile-long tunnel fitted with lights and ventilation. Making a break for it required creative thinking, long-term planning and perseverance – essential skills similar to those needed to achieve success in big business. 

D. While Devin Liddell, who heads brand strategy for Seattle-based design consultancy,Teague, condemns the violence and other illegal activities he became curious as to how criminal groups endure. Some cartels stay in business despite multiple efforts by law enforcement on both sides of the US border and millions of dollars from international agencies to shut them down. Liddell genuinely believes there’s a lesson in longevity here. One strategy he underlined was how the bad guys respond to change. In order to bypass the border between Mexico and the US, for example, the Sinaloa cartel went to great lengths. It built a vast underground tunnel, hired family members as border agents and even used a catapult to circumvent a high-tech fence. 

E. By contrast, many legitimate businesses fail because they hesitate to adapt quickly tochanging market winds. One high-profile example is movie and game rental company 

Blockbuster, which didn’t keep up with the market and lost business to mail order video rentals and streaming technologies. The brand has all but faded from view. Liddell argues the difference between the two groups is that criminal organisations often have improvisation encoded into their daily behaviour, while larger companies think of innovation as a set process. “This is a leadership challenge,” said Liddell. “How well companies innovate and organise is a reflection of leadership.” 

Left-field thinking 

F. Cash-strapped start-ups also use unorthodox strategies to problem solve and build theirbusinesses up from scratch. This creativity and innovation are often borne out of necessity, such as tight budgets. Both criminals and start-up founders “question authority, act outside the system and see new and clever ways of doing things,” said Goodman. “Either they become Elon Musk or El Chapo.” And, some entrepreneurs aren’t even afraid to operate in legal grey areas in their effort to disrupt the marketplace. The co-founders of music streaming service Napster, for example, knowingly broke music copyright rules with their first online file sharing service, but their technology paved the way for legal innovation as regulators caught up. 

G. Goodman and others believe thinking hard about problem-solving before worryingabout restrictions could prevent established companies from falling victim to rivals less constrained by tradition. In their book The Misfit Economy, Alexa Clay and Kyra Maya Phillips examine how individuals can apply that mindset to become more innovative and entrepreneurial within corporate structures. They studied not just violent criminals like Somali pirates, but others who break the rules in order to find creative solutions to their business problems, such as people living in the slums of Mumbai or computer hackers. They picked out five common traits among this group: the ability to hustle, pivot, provoke, hack and copycat. 

H. Clay gives a Saudi entrepreneur named Walid Abdul-Wahab as a prime example.Abdul-Wahab worked with Amish farmers to bring camel milk to American consumers even before US regulators approved it. Through perseverance, he eventually found a network of Amish camel milk farmers and started selling the product via social media. Now his company, Desert Farms, sells to giant mainstream retailers like Whole Foods Market. Those on the fringe don’t always have the option of traditional, corporate jobs and that forces them to think more creatively about how to make a living, Clay said. They must develop grit and resilience in order to last outside the cushy confines of cubicle life. “In many cases, scarcity is the mother of invention,” Clay said. 

Questions 14-21. Reading Passage 2 has eight paragraphs A-H.  Match the headings below with the paragraphs. Write the correct letter, A-H, in boxes 14-21 on your answer sheet. 

14. Jailbreak with creative thinking ____________

15. Five common traits among rule-breakers ____________

16. Comparison between criminals and traditional businessmen___________

17.Can drug baron’s e-space teach legitimate corporations? ___________

18.Great entrepreneur ___________ .

19.How criminal groups deceive the law ___________.

20.The difference between legal and illegal organizations ___________ .

21.Similarity between criminals and start-up founders __________ .

Questions 22–25. Complete the sentences below. Write ONLY ONE WORD from the passage for each answer. Write your answers in boxes 22–25 on your answer sheet. 

22. To escape from prison, Joaquin Guzman had to use such traits as creative thinking, long-term planning and _________.

23. The Sinaloa cartel built a grand underground tunnel and even used a __________ to avoidthe fence.

24. The main difference between the two groups is that criminals, unlike large corporations,often have _________ encoded into their daily life.

25. Due to being persuasive, Walid Abdul-Wahab found a ________ of Amish camel milkfarmers.

Question 26. Choose the correct letter, A, B, C or D. 

26. The main goal of this article is to:

A. Show different ways of illegal activity

B. Give an overview of various criminals and their gangs

C. Draw a comparison between legal and illegal business, providing examples

D. Justify criminals with creative thinking bottom of the form

Reading Passage 3.

You should spend about 20 minutes on Questions 28–40, which are based on Reading Passage 3 below. 

As More Tech Startups Stay Private, So Does the Money 

A. Not long ago, if you were a young, brash technologist with a world-conquering start-upidea, there was a good chance you spent much of your waking life working toward a singlebusiness milestone: taking your company public. Though luminaries of the tech industry havealways expressed scepticism and even hostility toward the finance industry, tech’s dirtysecret was that it looked to Wall Street and the ritual of a public offering for affirmation —not to mention wealth. But something strange has happened in the last couple of years: Theinitial public offering of stock has become déclassé. For start-up entrepreneurs and theiremployees across Silicon Valley, an initial public offering is no longer the main goal. Instead,many founders talk about going public as a necessary evil to be postponed as long as possiblebecause it comes with more problems than benefits.

B. “If you can get $200 million from private sources, then yeah, I don’t want my companyunder the scrutiny of the unwashed masses who don’t understand my business,” saidDanielle Morrill, the chief executive of Mattermark, a start-up that organizes and sellsinformation about the start-up market. “That’s actually terrifying to me.” Silicon Valley’ssudden distaste for the I.P.O. — rooted in part in Wall Street’s scepticism of new tech stocks— may be the single most important psychological shift underlying the current tech boom.Staying private affords start-up executives the luxury of not worrying what outsiders thinkand helps them avoid the quarterly earnings treadmill. It also means Wall Street is doingwhat it failed to do in the last tech boom: using traditional metrics like growth andprofitability to price companies. Investors have been tough on Twitter, for example, becauseits user growth has slowed. They have been tough on Box, the cloud-storage company thatwent public last year because it remains unprofitable. And the e-commerce company Zulily,which went public last year, was likewise punished when it cut its guidance for future sales.

C. Scott Kupor, the managing partner at the venture capital firm Andreessen Horowitz, andhis colleagues said in a recent report that despite all the attention start-ups have received inrecent years, tech stocks are not seeing unusually high valuations. In fact, their share of theoverall market has remained stable for 14 years, and far off the peak of the late 1990s. Thatunwillingness to cut much slack to young tech companies limits risk for regular investors. Ifthe bubble pops, the unwashed masses, if that’s what we are, aren’t as likely to get washedout. Private investors, on the other hand, are making big bets on so-called unicorns — theSilicon Valley jargon for start-up companies valued at more than a billion dollars. If any of those unicorns flops, most Americans will escape unharmed, because losses will be confined to venture capitalists and hedge funds that have begun to buy into tech start-ups, as well as tech founders and their employees. 

D. The reluctance — and sometimes inability — to go public is spurring the unicorns. By relying on private investors for a longer period of time, start-ups get more runway to figure out sustainable business models. To delay their entrance into the public markets, firms like Airbnb, Dropbox, Palantir, Pinterest, Uber and several other large start-ups are raising hundreds of millions, and in some cases billions, that they would otherwise have gained through an initial public offering. “These companies are going public, just in the private market,” Dan Levitan, the managing partner of the venture capital firm Maveron, told me recently. He means that in many cases, hedge funds and other global investors that would have bought shares in these firms after an I.P.O. are deciding to go into late-stage private rounds. There is even an oxymoronic term for the act of obtaining private money in place of a public offering: It’s called a “private I.P.O.” 

E. The delay in I.P.O.s has altered how some venture capital firms do business. Rather than waiting for an initial offering, Maveron, for instance, says it now sells its stake in a start-up to other, larger private investors once it has made about 100 times its initial investment. It is the sort of return that once was only possible after an I.P.O. But there is also a downside to the new version to initial offerings. When the unicorns do eventually go public and begin to soar — or whatever it is that fantastical horned beasts tend to do when they’re healthy — the biggest winners will be the private investors that are now bearing most of the risk. It used to be that public investors who got in on the ground floor of an initial offering could earn historic gains. If you invested $1,000 in Amazon at its I.P.O. in 1997, you would now have nearly $250,000. If you had invested $1,000 in Microsoft in 1986, you would have close to half a million. Public investors today are unlikely to get anywhere near such gains from tech I.P.O.s. By the time tech companies come to the market, the biggest gains have already been extracted by private backers. 

F. Just 53 technology companies went public in 2014, which is around the median since 1980, but far fewer than during the boom of the late 1990s and 2000, when hundreds of tech companies went public annually, according to statistics maintained by Jay Ritter, a professor of finance at the University of Florida. Today’s companies are also waiting for longer. In 2014, the typical tech company hitting the markets was 11 years old, compared with a median age of seven years for tech I.P.O.s since 1980. Over the last few weeks, I’ve asked several founders and investors why they’re waiting; few were willing to speak on the record about their own companies, but their answers all amounted to “What’s the point?” Initial public offerings were also ways to compensate employees and founders who owned lots of stock, but there are now novel mechanisms — such as selling shares on a secondary market — for insiders to cash in on some of their shares in private companies. Still, some observers cautioned that the new trend may be a bad deal for employees who aren’t given much information about the company’s performance. 

G. “One thing employees may be confused about is when companies tell them, ‘We’re basically doing a private I.P.O.,’ it might make them feel like there’s less risk than there really is,” said Ms.Morrill of Mattermark. But she said it was hard to persuade people that their paper gains may never materialize. “The Kool-Aid is really strong,” she said. If the delay in I.P.O.s becomes a normal condition for Silicon Valley, some observers say tech companiesmay need to consider new forms of compensation for workers. “We probably need tofundamentally rethink how do private companies compensate employees because that’sgoing to be an issue,” said Mr.Kupor, of Andreessen Horowitz.

H. During a recent presentation for Andreessen Horowitz’s limited partners — the institutionsthat give money to the venture firm — Marc Andreessen, the firm’s co-founder, told thejournalist Dan Primack that he had never seen a sharper divergence in how investors treatpublic- and private-company chief executives. “They tell the public C.E.O, ‘Give us the moneyback this quarter,’ and they tell the private C.E.O., ‘No problem, go for 10 years,’ ”Mr.Andreessen said. At some point, this tension will be resolved. “Private valuations will notforever be higher than public valuations,” said Mr. Levitan, of Maveron. “So the question is,will private markets capitulate and go down or will public markets go up?” If the privateinvestors are wrong, employees, founders and a lot of hedge funds could be in for areckoning. But if they’re right, it will be you and me wearing the frown — the public investorswho missed out on the next big thing.

Questions 28–31. Choose the correct letter, A, B, C or D. Write the correct letter in boxes 28–31 on your answer sheet. 

28. How many funds would you gain by now, if you had invested 1000$ in theAmazon in 1997?

A. 250,000$

B. close to 500,000$

C. It is not stated in the text

D. No funds

29. Nowadays founders talk about going public as a:

A. necessity

B. benefit

C. possibility

D. profit

30. In which time period was the biggest number of companies going public?

A. the early 1990s

B. the late 1900s and 2000s

C. The 1980s

D. the late 1990s

31. According to the text, which of the following is true?

A. Private valuations maybe forever higher than public ones.

B. Public valuations eventually will become even less valuable.

C. The main question is whether the public market increase or the private market decrease.

D. The pressure might last for a long time.

Questions 32–36. Complete the sentences below.  Write ONLY ONE WORD from the passage for each answer. Write your answers in boxes 32–36 on your answer sheet. 

32. Skepticism was always expected by the ___________of tech industry.

33. The new aversion to initial offerings has its ___________.

34. Selling shares on a secondary market is considered a ___________ mechanism.

35. Workers’ compensation might be an ___________.

36. The public investors who failed to participate in the next big thing might be the ones wearing the___________.

Questions 37–40. Do the following statements agree with the information in the IELTS reading text? In boxes 37–40 on your answer sheet, write 

TRUE, if the statement agrees with the information 

FALSE, if the statement contradicts the information 

NOT GIVEN, if there is no information on this 

37. Private investors are bearing most of the risk.

38. Not many investors were willing to speak on the record.

39. The typical tech company hitting the markets in the 1990s was 5 years old.

40. Marc Andreessen, the firm’s co-founder, expressed amazement with divergence in how investorstreat the public.

How did it go? Please share your feedback in the comment section below:
Do you need a copy of this test? Download PDF

Show answers
Aphantasia

1.         False

2.         True

3.         Not given

4.         True

5.         True

6.         Not given

7.         True

8.         False

9.         Vibrant

10.       Polar/opposite

11.       Grainy picture

12.       Adamant

13.       Imagery hovering

14.       C

15.       G

16.       B

17.       A

18.       H

19.       D

20.       E

21.       F

22.       Perseverance

23.       Catapult

24.       Improvisation

25.       Network

26.       C

27.       True

28.       False

29.       Not given

30.       True

31.       False

32.       A

33.       C

34.       A

35.       B

36.       Commercially abroad

37.       Subscription channels

38.       Icky

39.       More ambitious

40.       Speculation

Ant Intelligence

Reading passage 1

When we think of intelligent members of the animal kingdom, the creatures that spring immediately to mind are apes and monkeys. But in fact the social lives of some members of the insect kingdom are sufficiently complex to suggest more than a hint of intelligence. Among these, the world of the ant has come in for considerable scrutiny lately, and the idea that ants demonstrate sparks of cognition has certainly not been rejected by those involved in these investigations.

Ants store food, repel attackers and use chemical signals to contact one another in case of attack. Such chemical communication can be compared to the human use of visual and auditory channels (as in religious chants, advertising images and jingles, political slogans and martial music) to arouse and propagate moods and attitudes. The biologist Lewis Thomas wrote Ants are so much like human beings as to be an embarrassment. They farm fungi, raise aphids as livestock, launch armies to war, use chemical sprays to alarm and confuse enemies, capture slaves, engage in child labour, exchange information ceaselessly. They do everything but watch television.

However, in ants there is no cultural transmission – everything must be encoded in the genes – whereas in humans the opposite is true. Only basic instincts are carried in the genes of a newborn baby, other skills being learned from others in the community as the child grows up. It may seem that this cultural continuity gives us a huge advantage over ants. They have never mastered fire nor progressed. Their fungus farming and aphid herding crafts are sophisticated when compared to the agricultural skills of humans five thousand-years ago but have been totally overtaken by modem human agribusiness. Or have they? The farming methods of ants are at least sustainable. They do not ruin environments or use enormous amounts of energy. Moreover, recent evidence suggests that the crop farming of ants may be more sophisticated and adaptable than was thought.

Ants were farmers fifty million years before humans were. Ants can’t digest the cellulose in leaves – but some fungi can. The ants therefore cultivate these fungi in their nests, bringing them leaves to feed on, and then use them as a source of food. Farmer ants secrete antibiotics to control other fungi that might act as ‘weeds’, and spread waste to fertilise the crop.

It was once thought that the fungus that ants cultivate was a single type that they had propagated, essentially unchanged from the distant past. Not so. Ulrich Mueller of Maryland and his colleagues genetically screened 862 different types of fungi taken from ants’ nests. These turned out to be highly diverse: it seems that ants are continually domesticating new species. Even more impressively, DNA analysis of the fungi suggests that the ants improve or modify the fungi by regularly swapping and sharing strains with neighboring ant colonies.

Whereas prehistoric man had no exposure to urban lifestyles – the forcing house, of intelligence – the evidence suggests that ants have lived in urban settings for close on a hundred million years, developing and maintaining underground cities of specialised chambers and tunnels.

When we survey Mexico City, Tokyo, Los Angeles, we are amazed at what has been accomplished by humans. Yet Hoelldobler and Wilson’s magnificent work for ant lovers, the Ants, describes a supercolony of the ant Formica yessensis on the Ishikari Coast of Hokkaido. This ‘megalopolis’ was reported to be composed of 360 million workers and a million queens living in 4,500 interconnected nests across a territory of 2.7 square kilometers.

Such enduring and intricately meshed levels of technical achievement outstrip by far anything achieved by our distant ancestors. We hail as masterpieces the cave paintings in southern France and elsewhere, dating back some 20,000 years. Ant societies existed in something like their present form more than seventy million years ago. Beside this, prehistoric man looks technologically primitive. Is this then some kind of intelligence, albeit of a different kind?

Research conducted at Oxford, Sussex and Zurich Universities has shown that when; desert ants return from a foraging trip, they navigate by integrating bearings and distances, which they continuously update their heads. They combine the evidence of visual landmarks with a mental library of local directions, all within a framework which is consulted and updated. So ants can learn too.

And in a twelve-year programme of work, Ryabko and Reznikova have found evidence that ants can transmit very complex messages. Scouts who had located food in a maze returned to mobilise their foraging teams. They engaged in contact sessions, at the end of which the scout was removed in order to observe what her team might do. Often the foragers proceeded to the exact spot in the maze where the food had been. Elaborate precautions were taken to prevent the foraging team using odour clues. Discussion now centers on whether the route through the maze is communicated as a ‘left- right sequence of turns or as a ‘compass bearing and distance’ message.

During the course of this exhaustive study, Reznikova has grown so attached to her laboratory ants that she feels she knows them as individuals – even without the paint spots used to mark them. It’s no surprise that Edward Wilson, in his essay, ‘In the company of ants’, advises readers who ask what to do with the ants in their kitchen to: ‘Watch where you step. Be careful of little lives.’

Questions 1-6

Do the following statements agree with the information given in Reading Passage 1? In boxes 1-6 on your answer sheet, write:

TRUE              if the statement agrees with the information

FALSE           if the statement contradicts the information

NOT GIVEN if there is no information on this

  1. Ants use the same channels of communication as humans do.
  2. City life is one factor that encourages the development of intelligence.
  3. Ants can build large cities more quickly than humans do.
  4. Some ants can find their way by making calculations based on distance and position.
  5. In one experiment, foraging teams were able to use their sense of smell to find food.
  6. The essay. ‘In the company of ants’ explores ant communication.
Questions 7-13

Complete the summary using the list of words, A-O, below. Write the correct letter, A-O, in boxes 7-13 on your answer sheet.

Ants as farmers

Ants have sophisticated methods of farming, including herding livestock and growing crops, which are in many ways similar to those used in human agriculture. The ants cultivate a large number of different species of edible fungi which convert (7)…………..… into a form which they can digest. They use their own natural (8)…………….…  as weed-killers and also use unwanted materials as (9)……………….. Genetic analysis shows they constantly upgrade these fungi by developing new species and by (10)…………….. species with neighboring ant colonies. In fact, the farming methods of ants could be said to be more advanced than human agribusiness, since they use (11)………………… methods, they do not affect the (12)……………… and do not waste (13)……………………

Population Movement and Genetics

A Study of the origins and distribution of human populations used to be based on archaeological and fossil evidence. A number of techniques developed since the 1950s, however, have placed the study of these subjects on a sounder and more objective footing. The best information on early population movements is now being obtained from the ‘archaeology of the living body’, the clues to be found in genetic material.

B Recent work on the problem of when people first entered the Americas is an example of the value of these new techniques. North-east Asia and Siberia have long been accepted as the launching ground for the first human colonisers of the New World*. But was there one major wave of migration across the Bering Strait into the Americas, or several? And when did this event, or events, take place? In recent years, new clues have come from research into genetics, including the distribution of genetic markers in modern Native Americans.

C An important project, led by the biological anthropologist Robert Williams, focused on the variants (called Gm allotypes) of one particular protein – immunoglobin G — found in the fluid portion of human blood. All proteins ‘drift’, or produce variants, over the generations, and members of an interbreeding human population will share a set of such variants. Thus, by comparing the Gm allotypes of two different populations (e.g. two Indian tribes), one can establish their genetic ‘distance’, which itself can be calibrated to give an indication of the length of time since these populations last interbred.

D Williams and his colleagues sampled the blood of over 5,000 American Indians in western North America during a twenty- year period. They found that their Gm allotypes could be divided into two groups, one of which also corresponded to the genetic typing of Central and South American Indians. Other tests showed that the Inuit (or Eskimo) and Aleut formed a third group. From this evidence it was deduced that there had been three major waves of migration across the Bering Strait. The first, Paleo-lndian, wave more than 15,000 years ago was ancestral to all Central and South American Indians. The second wave, about 14,000-12,000 years ago, brought Na-Dene hunters, ancestors of the Navajo and Apache (who only migrated south from Canada about 600 or 700 years ago). The third wave, perhaps 10,000 or 9,000 years ago, saw the migration from North-east Asia of groups ancestral to the modern Eskimo and Aleut.

E How far does other research support these conclusions? Geneticist Douglas Wallace has studied mitochondrial DNA in blood samples from three widely separated Native American groups: Pima- Papago Indians in Arizona, Maya Indians on the Yucatdn peninsula, Mexico, and Ticuna Indians in the Upper Amazon region of Brazil. As would have been predicted by Robert Williams’s work, all three groups appear to be descended from the same ancestral (Paleo-lndian) population.

F There are two other kinds of research that have thrown some light on the origins of the Native American population; they involve the study of teeth and of languages. The biological anthropologist Christy Turner is an expert in the analysis of changing physical characteristics in human teeth. He argues that tooth crowns and roots have a high genetic component, minimally affected by environmental and other factors. Studies carried out by Turner of many thousands of New and Old World specimens, both ancient and modern, suggest that the majority of prehistoric Americans are linked to Northern Asian populations by crown and root traits such as incisor6 shoveling (a scooping out on one or both surfaces of the tooth), single-rooted upper first premolars6 and triple- rooted lower first molars.

According to Turner, this ties in with the idea of a single Paleo-lndian migration out of North Asia, which he sets at before 14,000 years ago by calibrating rates of dental micro-evolution. Tooth analyses also suggest that there were two later migrations of Na-Denes and Eskimo- Aleut.

G The linguist Joseph Greenberg has, since the 1950s, argued that all Native American languages belong to a single ‘Amerind’ family, except for Na-Dene and Eskimo-Aleut – a view that gives credence to the idea of three main migrations. Greenberg is in a minority among fellow linguists, most of whom favour the notion of a great many waves of migration to account for the more than 1,000 languages spoken at one time by American Indians. But there is no doubt that the new genetic and dental evidence provides strong backing for Greenberg’s view. Dates given for the migrations should nevertheless be treated with caution, except where supported by hard archaeological evidence.

Questions 14-19

Reading Passage 2 has seven sections, A-G. Choose the correct headings for sections A-F from the list of headings below. Write the correct number, i-x, in boxes 14-19 on your answer sheet.

Questions 14-19

Reading Passage 2 has seven sections, A-G. Choose the correct headings for sections A-F from the list of headings below. Write the correct number, i-x, in boxes 14-19 on your answer sheet.
List of Headings
i The results of the research into blood-variants
ii Dental evidence
iii Greenberg’s analysis of the dental and linguistic evidence
iv Developments in the methods used to study early population movements
v Indian migration from Canada to the U.S.A.
vi Further genetic evidence relating to the three-wave theory
vii Long-standing questions about prehistoric migration to America
viii Conflicting views of the three-wave theory, based on non-genetic Evidence
ix Questions about the causes of prehistoric migration to America
x How analysis of blood-variants measures the closeness of the relationship between different populations
14 Passage A
15 Passage B
16 Passage C
17 Passage D
18 Passage E
19 Passage F
Example Section G viii

Questions 20 and 21

The discussion of Williams’s research indicates the periods at which early people are thought to have migrated along certain routes. There are six routes, A-F, marked on the map below.

Complete the form below. Write the correct letter A-F in boxes 20-21 on your answer sheet.

RoutePeriod (number of years ago)
20………………………..15,000 or more
21………………………..600 to 700
Questions 22-25

Reading Passage 2 refers to the three-wave theory of early migration to the Americas. It also suggests in which of these three waves the ancestors of various groups of modern native Americans first reached the continent.

Classify the groups named in the table below as originating from

A the first wave

B the second wave

C the third wave


Write the correct letter. A. B or C. in boxes 22-25 on your answer sheet.

Name of groupWave number
Inuit22…………………..
Apache23…………………..
Pima-Papago24…………………..
Ticuna25…………………..

Question 26. Choose the correct letter A, B, Cor D. Write the correct letter in box 26 on your answer sheet.
Christy Turner’s research involved the examination of

A teeth from both prehistoric and modern Americans and Asians
B thousands of people who live in either the New or the Old World
C dental specimens from the majority of prehistoric Americans
D the eating habits of American and Asian populations

Reading Passage 3

Forests are one of the main elements of our natural heritage. The decline of Europe’s forests over the last decade and a half has led to an increasing awareness and understanding of the serious imbalances which threaten them. European countries are becoming increasingly concerned by major threats to European forests, threats which know no frontiers other than those of geography or climate: air pollution, soil deterioration, the increasing number of forest fires and sometimes even the mismanagement of our woodland and forest heritage. There has been a growing awareness of the need for countries to get together to co-ordinate their policies. In December 1990, Strasbourg hosted the first Ministerial Conference on the protection of Europe’s forests. The conference brought together 31 countries from both Western and Eastern Europe. The topics discussed included the coordinated study of the destruction of forests, as well as how to combat forest fires and the extension of European research programs on the forest ecosystem. The preparatory work for the conference had been undertaken at two meetings of experts. Their initial task was to decide which of the many forest problems of concern to Europe involved the largest number of countries and might be the subject of joint action. Those confined to particular geographical areas, such as countries bordering the Mediterranean or the Nordic countries therefore had to be discarded. However, this does not mean that in future they will be ignored.

As a whole, European countries see forests as performing a triple function: biological, economic and recreational. The first is to act as a ‘green lung’ for our planet; by means of photosynthesis, forests produce oxygen through the transformation of solar energy, thus fulfilling what for humans is the essential role of an immense, non-polluting power plant. At the same time, forests provide raw materials for human activities through their constantly renewed production of wood. Finally, they offer those condemned to spend five days a week in an urban environment an unrivalled area of freedom to unwind and take part in a range of leisure activities, such as hunting, riding and hiking. The economic importance of forests has been understood since the dawn of man – wood was the first fuel. The other aspects have been recognised only for a few centuries but they are becoming more and more important. Hence, there is a real concern throughout Europe about the damage to the forest environment which threatens these three basic roles.

The myth of the ‘natural’ forest has survived, yet there are effectively no remaining ‘primary’ forests in Europe. All European forests are artificial, having been adapted and exploited by man for thousands of years. This means that a forest policy is vital, that it must transcend national frontiers and generations of people, and that it must allow for the inevitable changes that take place in the forests, in needs, and hence in policy. The Strasbourg conference was one of the first events on such a scale to reach this conclusion. A general declaration was made that ‘a central place in any ecologically coherent forest policy must be given to continuity over time and to the possible effects of unforeseen events, to ensure that the full potential of these forests is maintained.

That general declaration was accompanied by six detailed resolutions to assist national policy-making. The first proposes the extension and systematisation of surveillance sites to monitor forest decline. Forest decline is still poorly understood but leads to the loss of a high proportion of a tree’s needles or leaves. The entire continent and the majority of species are now affected: between 30% and 50% of the tree population. The condition appears to result from the cumulative effect of a number of factors, with atmospheric pollutants the principal culprits.

Compounds of nitrogen and sulphur dioxide should be particularly closely watched. However, their effects are probably accentuated by climatic factors, such as drought and hard winters, or soil imbalances such as soil acidification, which damages the roots. The second resolution concentrates on the need to preserve the genetic diversity of European forests. The aim is to reverse the decline in the number of tree species or at least to preserve the ‘genetic material’ of all of them. Although forest fires do not affect all of Europe to the same extent, the amount of damage caused the experts to propose as the third resolution that the Strasbourg conference consider the establishment of a European databank on the subject.

All information used in the development of national preventative policies would become generally available. The subject of the fourth resolution discussed by the ministers was mountain forests. In Europe, it is undoubtedly the mountain ecosystem which has changed most rapidly and is most at risk. A thinly scattered permanent population and development of leisure activities, particularly skiing, have resulted in significant long-term changes to the local ecosystems. Proposed developments include a preferential research program on mountain forests. The fifth resolution relaunched the European research network on the physiology of trees, called Eurosilva. Eurosilva should support joint European research on tree diseases and their physiological and biochemical aspects. Each country concerned could increase the number of scholarships and other financial support for doctoral theses and research projects in this area. Finally, the conference established the framework for a European research network on forest ecosystems. This would also involve harmonising activities in individual countries as well as identifying a number of priority research topics relating to the protection of forests. The Strasbourg conference’s main concern was to provide for the future. This was the initial motivation, one now shared by all 31 participants representing 31 European countries. Their final text commits them to on-going discussion between government representatives with responsibility for forests.

Questions 27-33

Do the following statements agree with the information given in Reading Passage 3? In boxes 27-33 on your answer sheet, write

TRUE                 if the statement agrees with the information

FALSE         if the statement contradicts the information

NOT GIVEN  if there is no information on this

27 Forest problems of Mediterranean countries are to be discussed at the next meeting of experts.
28 Problems in Nordic countries were excluded because they are outside the European Economic Community.
29 Forests are a renewable source of raw material.
30 The biological functions of forests were recognised only in the twentieth century.
31 Natural forests still exist in parts of Europe.
32 Forest policy should be limited by national boundaries.
33 The Strasbourg conference decided that a forest policy must allow for the possibility of change.

Questions 34-39

Look at the following statements issued by the conference.

Which SIX of the following statements. A-J, refer to the resolutions that were issued? Match the statements with the appropriate resolutions (Questions 34-39).

A All kinds of species of trees should be preserved.

B Fragile mountain forests should be given priority in research programs.

C The surviving natural forests of Europe do not need priority treatment.

D Research is to be better co-ordinate throughout Europe.

E Information on forest fires should be collected and shared.

F Loss Of leaves from trees should be more extensively and carefully monitored.

G Resources should be allocated to research into tree diseases.

H Skiing should be encouraged in thinly populated areas.

I Soil imbalances such as acidification should be treated with compounds of nitrogen and sulphur.
J Information is to be systematically gathered on any decline in the condition of forests.

34 Resolution 1

35 Resolution 2

36 Resolution 3

37 Resolution 4

38 Resolution 5

39 Resolution 6

Question 40. Choose the correct letter, A. B, C or D. Write the correct letter in box 40 on your answer sheet.

40  What is the best title for Reading Passage 3?

A The biological, economic and recreational role of forests

B Plans to protect the forests of Europe

C The priority of European research into ecosystems

D Proposals for a world-wide policy on forest management

How did it go? Please share your feedback in the comment section below:
Do you need a copy of this test? Download PDF

Show answers
Ant intelligence

1.         False

2.         True

3.         Not given

4.         True

5.         False

6.         Not given

7.         C

8.         M

9.         F

10.       D

11.       N

12.       O

13.       E

14.       Iv

15.       Vii

16.       X

17.       I

18.       Vi

19.       Ii

20.       E

21.       D

22.       C

23.       B

24.       A

25.       A

26.       A

27.       Not given

28.       False

29.       True

30.       False

31.       False

32.       False

33.       True

34.       J

35.       A

36.       E

37.       B

38.       G

39.       D

40.       B

Animal Minds: Parrot Alex IELTS Reading

Reading passage 1

A. In 1977 Irene Pepperberg, a recent graduate of Harvard University did something very bold. At a time when animals still were considered automatons, she set out to find what was on another creature’s mind by talking to it. She brought a one-year-old African grey parrot she named Alex into her lab to teach him to reproduce the sounds of the English language. “I thought if he learned to communicate, I could ask him questions about how he sees the world.”

B. When Pepperberg began her dialogue with Alex, who died last September at the age of 31, many scientists believed animals were incapable of any thought. They were simply machines, robots programmed to react to stimuli but lacking the ability to think or feel. Any pet owner would disagree. We see the love in our dogs’ eyes and know that, of course, they have thoughts and emotions. But such claims remain highly controversial. Gut instinct is not science, and it is all too easy to project human thoughts and feelings onto another creature. How, then, does a scientist prove that an animal is capable of thinking – that it can acquire information about the world and act on it? “That’s why I started my studies with Alex,” Pepperberg said. They were seated – she at her desk, he on top of his cage – in her lab, a windowless room about the size of a boxcar, at Brandeis University. Newspapers lined the floor; baskets of bright toys were stacked on the shelves. They were a team – and because of their work, the notion that animals can think is no longer so fanciful.

C. Certain skills are considered key signs of higher mental abilities: good memory, a grasp of grammar and symbols, self-awareness, understanding others’ motives, imitating others, and being creative. Bit by bit, in ingenious experiments, researchers have documented these talents in other species, gradually chipping away at what we thought made human beings distinctive while offering a glimpse of where our own abilities came from. Scrub jays know that other jays are thieves and that stashed food can spoil; sheep can recognize faces; chimpanzees use a variety of tools to probe termite mounds and even use weapons to hunt small mammals; dolphins can imitate human postures; the archerfish, which stuns insects with a sudden blast of water, can learn how to aim its squirt simply by watching an experienced fish perform the task and Alex the parrot turned out to be a surprisingly good talker.

D. Thirty years after the Alex studies began; Pepperberg and a changing collection of assistants were still giving him English lessons. The humans, along with two younger parrots, also served as Alex’s flock, providing the social input all parrots crave. Like any flock, this one – as small as it was – had its share of drama. Alex dominated his fellow parrots, acted huffy at times around Pepperberg, tolerated the other female humans, and fell to pieces over a male assistant who dropped by for a visit. Pepperberg bought Alex in a Chicago pet store where she let the store’s assistant pick him out because she didn’t want other scientists saying later that she’d particularly chosen an especially smart bird for her work. Given that Alex’s brain was the size of a shelled walnut, most researchers thought Pepperberg’s interspecies communication study would be futile.

E.“Some people actually called me crazy for trying this,” she said. “Scientists thought that chimpanzees were better subjects, although, of course, chimps can’t speak.” Chimpanzees, bonobos, and gorillas have been taught to use sign language and symbols to communicate with us, often with impressive results. The bonobo Kanzi, for instance, carries his symbol-communication board with him so he can “talk” to his human researchers, and he has invented combinations of symbols to express his thoughts. Nevertheless, this is not the same thing as having an animal look up at you, open his mouth, and speak. Under Pepperberg’s patient tutelage, Alex learned how to use his vocal tract to imitate almost one hundred English words, including the sounds for various foods, although he calls an apple a “beanery.” “Apples taste a little bit like bananas to him, and they look a little bit like cherries, Alex made up that word for them,” Pepperberg said.

F. It sounded a bit mad, the idea of a bird having lessons to practice, and willingly doing it. But after listening to and observing Alex, it was difficult to argue with Pepperberg’s explanation for his behaviours. She wasn’t handing him treats for the repetitious work or rapping him on the claws to make him say the sounds. “He has to hear the words over and over before he can correctly imitate them,” Pepperberg said, after pronouncing “seven” for Alex a good dozen times in a row. “I’m not trying to see if Alex can learn a human language,” she added. “That’s never been the point. My plan always was to use his imitative skills to get a better understanding of avian cognition.”

G. In other words, because Alex was able to produce a close approximation of the sounds of some English words, Pepperberg could ask him questions about a bird’s basic understanding of the world. She couldn’t ask him what he was thinking about, but she could ask him about his knowledge of numbers, shapes, and colours. To demonstrate, Pepperberg carried Alex on her arm to a tall wooden perch in the middle of the room. She then retrieved a green key and a small green cup from a basket on a shelf. She held up the two items to Alex’s eye. “What’s the same?” she asked. Without hesitation, Alex’s beak opened: “Color.” “What’s different?” Pepperberg asked. “Shape,” Alex said. His voice had the digitized sound of a cartoon character. Since parrots lack lips (another reason it was difficult for Alex to pronounce some sounds, such as ba), the words seemed to come from the air around him, as if a ventriloquist were speaking. But the words – and what can only be called the thoughts – were entirely his.

H. For the next 20 minutes, Alex ran through his tests, distinguishing colours, shapes, sizes, and materials (wool versus wood versus metal). He did some simple arithmetic, such as counting the yellow toy blocks among a pile of mixed hues. And, then, as if to offer final proof of the mind inside his bird’s brain, Alex spoke up. “Talk clearly!” he commanded, when one of the younger birds Pepperberg was also teaching talked with wrong pronunciation. “Talk clearly!” “Don’t be a smart aleck,” Pepperberg said, shaking her head at him. “He knows all this, and he gets bored, so he interrupts the others, or he gives the wrong answer just to be obstinate. At this stage, he’s like a teenager; he’s moody, and I’m never sure what he’ll do.”

Questions 1-6. Do the following statements agree with the information given in Reading Passage? In boxes 1-6 on your answer sheet, write
TRUE, if the statement agrees with the information
FALSE, if the statement contradicts the information
NOT GIVEN, if there is no information on this

1. Firstly, Alex has grasped quite a lot of vocabulary.
2. At the beginning of the study, Alex felt frightened in the presence of humans.
3. Previously, many scientists realized that animals possess the ability of thinking.
4. It has taken a long time before people get to know cognition existing in animals.
5. As Alex could approximately imitate the sounds of English words, he was capable of roughly answering Irene’s questions regarding the world.
6. By breaking in other parrots as well as producing the incorrect answers, he tried to be focused.

Questions 7-10. Complete the following summary of the paragraphs of Reading Passage, using NO MORE THAN THREE WORDS from the Reading Passage for each answer. Write your answers in boxes 7-10 on your answer sheet.

After the training of Irene, Parrot Alex can use his vocal tract to pronounce more than 7…………………, while other scientists believe that animals have no this advanced ability of thinking, they would rather teach 8…………………. Pepperberg clarified that she wanted to conduct a study concerning 9………………… but not to teach him to talk. The store’s assistant picked out a bird at random for her for the sake of avoiding other scientists saying that the bird is 10 ………………… afterwards.

Questions 11-13. Answer the questions 11-13 below.
Choose NO MORE THAN THREE WORDS AND/OR A NUMBER from the passage for each answer.

11. What did Alex reply regarding the similarity of the subjects showed to him?
12. What is the problem of the young parrots except for Alex?
13. To some extent, through the way, he behaved what we can call him


Reading Passage 2

Developing Courtiers

A. The Ecotourism Society defines ecotourism as “responsible travel to natural areas which conserves the environment and improves the welfare of local people”. It is recognized as being particularly conducive to enriching and enhancing the standing of tourism, on the basis that this form of tourism respects the natural heritage and local populations and are in keeping with the carrying capacity of the sites. Cuba is undoubtedly an obvious site for ecotourism, with its picturesque beaches, underwater beauty, countryside landscapes, and ecological reserves.

B. An educated population and improved infrastructure of roads and communications add to the mix. In the Caribbean region, Cuba is now the second most popular tourist destination. Ecotourism is also seen as an environmental education opportunity to heighten both visitors’ and residents’ awareness of environmental and conservation issues, and even to inspire conservation action. Ecotourism has also been credited with promoting peace, by providing opportunities for educational and cultural exchange. Tourists’ safety and health are guaranteed. Raul Castro, brother of the Cuban president, started this initiative to rescue the Cuban tradition of herbal medicine and provide natural medicines for its healthcare system. The school at Las Terrazas Eco-Tourism Community teaches herbal healthcare and children learn not only how to use medicinal herbs, but also to grow them in the school garden for teas, tinctures, ointments, and creams. In Cuba, ecotourism has the potential to alleviate poverty by bringing money into the economy and creating jobs. In addition to the environmental impacts of these efforts, the area works on developing community employment opportunities for locals, in conjunction with ecotourism.

C. In terms of South America, it might be the place which shows the shortcoming of ecotourism. Histoplasma capsulatum, a dimorphic fungus, is the most common endemic mycosis in the United States and is associated with exposure to bat or bird droppings. Most recently, outbreaks have been reported in healthy travellers who returned from Central and South America after engaging in recreational activities associated with spelunking, adventure tourism, and ecotourism. It is quite often to see tourists neglected sanitation while travelling. After engaging in high-risk activities, boots should be hosed off and clothing placed in airtight plastic bags for laundering. HIV- infected travellers should avoid risky behaviours or environments, such as exploring caves, particularly those that contain bat droppings.

D. Nowhere is the keen eye and intimate knowledge of ecotourism are more amidst this fantastic biodiversity, as we explore remote realms rich in wildlife rather than a nature adventure. A sustainable tour is significant for ecotourism, one in which we can grow hand in hand with nature and our community, respecting everything that makes us privileged. Travellers get great joy from every step that takes forward on this endless but exciting journey towards sustainability. The primary threats to South America’s tropical forests are deforestation caused by agricultural expansion, cattle ranching, fagging, oil extraction and spills, mining, illegal coca farming, and colonization initiatives. Deforestation has shrunk territories belonging to indigenous peoples and wiped out more than 90% of the population. Many are taking leading roles in sustainable tourism even as they introduce protected regions to more travellers.

E. In East Africa, significantly reducing such illegal hunting and allowing wildlife populations to recover would allow the generation of significant economic benefits through trophy hunting and potentially ecotourism. “Illegal hunting is an extremely inefficient use of wildlife resources because it fails to capture the value of wildlife achievable through alternative forms of use such as trophy hunting and ecotourism,” said Peter Lindsey, author of the new study. Most residents believed that ecotourism could solve this circumstance. They have a passion for local community empowerment, loves photography and writes to laud current local conservation efforts, create environmental awareness and promote ecotourism.

F. In Indonesia, ecotourism started to become an important concept from 1995, in order to strengthen the domestic travelling movement, the local government targeting the right markets is a prerequisite for successful
ecotourism. The market segment for Indonesian ecotourism consists of: (i) “The silent generation”, 55-64-year-old people who are wealthy enough, generally well-educated and have no dependent children, and can travel for four weeks; (ii) “The baby boom generation”, junior successful executives aged 35-54 years, who are likely to be travelling with their family and children (spending 2-3 weeks on travel) – travelling for them is a stress reliever; and (iii) the “X generation”, aged 18-29 years, who love to do ecotours as backpackers – they are generally students who can travel for 3-12 months with a monthly expenditure of US$300-500. It is suggested that the promotion of Indonesian ecotourism products should aim to reach these various cohorts of tourists. The country welcomes diverse levels of travellers.

G. On the other hand, ecotourism provides as many services as traditional tourism. Nestled between Mexico, Guatemala and the Caribbean Sea is the country of Belize. It is the wonderful place for Hamanasi honeymoon, a bottle of champagne upon arrival, three meals daily, private service on one night of your stay and a choice of adventures depending on the length of your stay. It also offers six-night and seven-night honeymoon packages. A variety of specially tailored tours, including the Brimstone Hill Fortress, and a trip to a neighbouring island. Guided tours include rainforest, volcano, and off-road plantation tours. Gregory Pereira, extremely knowledgeable and outgoing hiking and tour guide say the following about his tours: “All of our tours on St. Kitts include transportation by specially modified Land Rovers, a picnic of island pastries and local fruit, fresh tropical juices, CSR, a qualified island guide and a full liability insurance coverage for participants.

H. Kodai is an ultimate splendour spot for those who love being close to mother nature. They say every bird must sing its own throat while we say every traveller should find his own way out of variegated and unblemished paths of deep valleys and steep mountains. The cheese factory here exports a great quantity of cheese to various countries across the globe. It is located in the centre of the forest. Many travellers are attracted by the delicious cheese. Ecotourism is very famous for this different eating experience.

Questions 14-18

Use the information in the passage to match the place (listed A-D) with opinions or deeds below. Write the appropriate letters, A-D, in boxes 14-18 on your answer sheet.
NB: You may use any letter more than once.

14. a place to improve local education to help tourists
15. a place suitable for both rich and poor travellers
16. a place where could easily get fungus
17. a place taking a method to stop unlawful poaching
18. a place where the healthcare system is developed

A. Cuba
B. East Africa
C. South America
D. Indonesia

Questions 19-22. Use the information in the passage to match the companies (listed A-C) with or deeds below. Write the appropriate letters A, B, C or D in boxes 19-22 answer sheet.

Eating the local fruits at the same time find job opportunities in the community which is situated on the heart of the jungle with private and comfortable service

19. Visiting the cheese factory
20. Enjoying the honeymoon
21. Having the picnic while
22. The residents in Cuba could

Questions 23-26. Complete the following summary of the paragraphs of Reading Passage, using NO MORE THAN TWO WORDS from the Reading Passage for each answer. Write your answers in boxes 23-26 on your answer sheet.

Ecotourism is not nature 23…………………… but a 24……………… tour. The reason why South America promotes ecotourism is due to the destruction of 25 In addition, East Africa also encourages this kind of tourism for cutting the 26……………………in order to save wild animals.

Reading Passage 3

Classifying Societies

Although humans have established many types of societies throughout history sociologists and anthropologists tend to classify different societies according to the degree to which different groups within a society have unequal access to advantages such as resources, prestige or power, and usually refer to four basic types of societies. From least to most socially complex, they are clans, tribes, chiefdoms, and states.

Clan

These are small-scale societies of hunters and gatherers, generally of fewer than 100 people, who move seasonally to exploit wild (undomesticated) food resources. Most surviving hunter-gatherer groups are of this kind, such as the Hadza of Tanzania or the San of southern Africa. Clan members are generally kinsfolk, related by descent or marriage. Clans lack formal leaders, so there are no marked economic differences or disparities in status among their members.
Because clans are composed of mobile groups of hunter-gatherers, their sites consist mainly of seasonally occupied camps, and other smaller and more specialized sites. Among the latter are kill or butchery sites – locations where large mammals are killed and sometimes butchered – and work sites, where tools are made or other specific activities carried out. The base camp of such a group may give evidence of rather insubstantial dwellings or temporary shelters, along with the debris of residential occupation.

Tribe

These are generally larger than mobile hunter-gatherer groups, but rarely number more than a few thousand, and their diet or subsistence is based largely on cultivated plants and domesticated animals. Typically, they have settled farmers, but they may be nomadic with a very different, mobile economy based on the intensive exploitation of livestock. These are generally multi-community societies, with the individual communities integrated into the large society through kinship ties. Although some tribes have officials and even a “capital” or seat of government, such officials lack the economic base necessary for effective use of power.
The typical settlement pattern for tribes is one of settled agricultural homesteads or villages. Characteristically, no one settlement dominates any of the others in the region. Instead, the archaeologist finds evidence for isolated, permanently occupied houses or for permanent villages. Such villages may be made up of a collection of free- standing houses, like those of the first farms of the Danube valley in Europe. Or they may be clusters of buildings grouped together, for example, the pueblos of the American Southwest, and the early farming village or the small town of Catalhoyuk in modern Turkey.

Chiefdom

These operate on the principle of ranking-differences in social status between people. Different lineages (a lineage is a group claiming descent from a common ancestor) are graded on a scale of prestige, and the senior lineage, and hence the society as a whole, is governed by a chief. Prestige and rank are determined by how closely related one is to the chief, and there is no true stratification into classes. The role of the chief is crucial.
Often, there is local specialization in craft products, and surpluses of these and of foodstuffs are periodically paid as an obligation to the chief. He uses these to maintain his retainers and may use them for redistribution to his subjects. The chiefdom generally has a centre of power, often with temples, residences of the chief and his retainers, and craft specialists. Chiefdoms vary greatly in size, but the range is generally between about 5000 and 20,000 persons.

Early State

These preserve many of the features of chiefdoms, but the ruler (perhaps a king or sometimes a queen) has explicit authority to establish laws and also to enforce them by the use of a standing army. Society no longer depends totally upon kin relationships: it is now stratified into different classes. Agricultural workers and the poorer urban dwellers form the lowest classes, with the craft specialists above, and the priests and kinsfolk of the ruler higher still. The functions of the ruler are often separated from those of the priest: the palace is distinguished from the temple. The society is viewed as a territory owned by the ruling lineage and populated by tenants who have an obligation to pay taxes. The central capital houses a bureaucratic administration of officials; one of their principal purposes is to collect revenue (often in the form of taxes and tolls) and distribute it to government, army and craft specialists. Many early states developed complex redistribution systems to support these essential services.

This rather simple social typology, set out by Elman Service and elaborated by William Sanders and Joseph Marino, can be criticized, and it should not be used unthinkingly. Nevertheless, if we are seeking to talk about early societies, we must use words and hence concepts to do so. Service categories provide a good framework to help organize our thoughts.

Questions 27-33
Do the following statements agree with the information given in Reading Passage? In boxes 27-33 on your answer sheet, write
TRUE FALSE NOT GIVEN

27. There’s little economic difference between members of a clan.
28. The farmers of a tribe grow a wide range of plants.
29. One settlement is more important than any other settlement in a tribe.
30. A member’s status in a chiefdom is determined by how much land he owns.
31. There are people who craft goods in chiefdoms. 32. The king keeps the order of a state by keeping a military.
33. Bureaucratic officers receive higher salaries than other members.

Questions 34-39

Answer the questions below. NO MORE THAN TWO WORDS

34. What is made at the clan work sites?
35. What is the other way of life for tribes besides settled farming?
36. How are Catalhoyuk’s housing units arranged?
37. What does a chief give to his subjects as rewards besides crafted goods?
38. What is the largest possible population of a chiefdom?
39. Which group of people is at the bottom of an early state but higher than the farmers?


40. Write the correct letter on the answer sheet.
In early states, what distinguishes the palace from the temple in terms of function and role?
A) The palace is the religious center, while the temple is the seat of government.
B) The palace is where craft specialists work, while the temple houses the king or queen.
C) The palace is responsible for tax collection, while the temple enforces laws.
D) The palace is where the ruler resides, while the temple is the religious center.

View answers- Animal Minds: Parrot Alex

How did it go? Please share your feedback in the comment section below:

Alternative Transportation IELTS reading

SECTION 1

Transportation is a major issue in urban areas around the world. Rising fuel costs, environmental problems, and traffic-clogged roads are some of the concerns that have led people to consider alternative forms of transportation.

Fuel-efficient cars and cars that run on alternative sources of energy are receiving increasing interest as people become more concerned about the costs of using gasoline. These costs include not only the ever increasing price of filling up a car’s fuel tank but also the environmental costs of emitting huge amounts of car exhaust into the atmosphere. Climate change is an issue of global concern. Closer to home, cities have to consider the effects on the health of their citizens. Car emissions have been linked to a range of health problems, particularly respiratory problems. For example, studies have linked childhood asthma and stunted lung growth to exposure to car exhaust in the air. Research has also made connections between car emissions and heart disease, certain cancers, and immune system problems.

The popularity of smaller, more fuel efficient cars is on the rise. Hybrid vehicles are also becoming more common. These cars have two engines— one that is battery powered and one that is gasoline powered. The batterypowered engine gets the car moving from a standstill. Once the car reaches a certain speed, the gasoline engine, which is more efficient at higher speeds, takes over to keep the car moving. There is also a growing interest in cars that are completely battery powered. These are cars that would be plugged into an electric outlet to recharge when not in use. Many consider such vehicles to be the car of the future. However, as long as the electricity is generated by coal-burning plants, as is often the case, these cars cannot be considered as using clean energy. Solar cars and hydrogen cars are other “clean” technologies that are receiving attention and hopes for the future.

Car emissions are the most serious source of concern, but the sheer number of vehicles on the road—over 250 million in the United States alone and over one billion worldwide—has other repercussions, as well. The roads and highways that are built to accommodate the growing number of cars in use are a source of pollution themselves. Ground that is covered with pavement cannot absorb rainwater, thus motor oil and other pollutants are washed off the roads and into lakes, rivers, and the ocean. Chemicals, herbicides, concrete, asphalt, paint, and other materials that are used during road construction also contribute to environmental pollution.

Personal convenience and health are also affected. While private cars are seen as a convenient way to get from place to place, crowded roads mean traffic moves much more slowly, making it difficult to travel, especially during “rush hour” periods. And people who spend hours each day sitting in cars stuck in traffic are not standing up, moving around, or getting any sort of exercise, a situation that can lead to a variety of health problems. Thus, in addition to developing passenger cars that run on alternative sources of fuel, we also need to look at alternative forms of transportation. These would include walking, bicycle riding, carpooling, and various types of public transportation. The benefits of walking and cycling are obvious. They cause no pollution and improve physical health. Car pools—several people sharing a ride in a private car—mean fewer cars on the road and allow the riders to share the expenses involved. Public transportation— buses, subways, commuter trains—has many benefits, as well. For one, it may provide users with opportunities for physical exercise as people have to get from their homes to the bus stops and train stations, and this is often done on foot. There are also mental health benefits, as relaxing on a train or bus while reading the newspaper or listening to music is a good deal less stressful than driving one’s own car through rush hour traffic. All of these forms of transportation decrease the number of cars on the roads and greatly reduce emissions. Looking toward the future, cities need to pay as much attention, or more, to public transportation and to accommodating walkers and cyclists as they do to building roads and accommodating drivers of passenger cars.

Questions 1-5 The list below shows some problems that are associated with the use of private cars.

Which FIVE of these problems are mentioned in the article?

A Social isolation
B High maintenance costs
C Air pollution
D Noise pollution
E Traffic congestion
F Stress
G Lack of parking space
H Rising price of gasoline
I Reduced opportunities for physical exercise

Questions 6-13. Do the following statements agree with the views of the writer in the passage? In boxes 6-13 on your answer sheet write

YES if the statement agrees with the views of the writer
NO if the statement contradicts the views of the writer
NOT GIVEN if it is impossible to say what the writer thinks about this

6 Car emissions can contribute to illnesses of the respiratory system.

7 Cars are the largest source of environmental pollution in the modern world.

8 People are becoming more interested in hybrid cars.

9 Electric cars don’t pollute the environment.

10 Solar-powered cars are currently too expensive for the average person to own.

11 Roads and highways contribute to water pollution.

12 Bicycle riding has health benefits.

13 Car pools can reduce individuals’ transportation costs.

SECTION 2

Less Television, Less Violence and Aggression

Cutting back on television, videos, and video games reduces acts of aggression among schoolchildren, according to a study by Dr. Thomas Robinson and others from the Stanford University School of Medicine. The study, published in the January 2001 issue of the Archives of Pediatric and Adolescent Medicine, found that third- and fourth-grade students who took part in a curriculum to reduce their TV video, and video game use engaged in fewer acts of verbal and physical aggression than their peers. The study took place in two similar San Jose, California, elementary schools. Students in one school underwent an 18-lesson, 6-month program designed to limit their media usage, while the others did not. Both groups of students had similar reports of aggressive behavior at the beginning of the study. After the six-month program, however, the two groups had very real differences.

The students who cut back on their TV time engaged in six fewer acts of verbal aggression per hour and rated 2.4 percent fewer of their classmates as aggressive after the program. Physical acts of violence, parental reports of aggressive behavior, and perceptions of a mean and scary world also decreased, but the authors suggest further study to solidify these results. Although many studies have shown that children who watch a lot of TV are more likely to act violently, this report further verifies that television, videos, and video games actually cause the violent behavior, and it is among the first to evaluate a solution to the problem.

Teachers at the intervention school included the program in their existing curriculum. Early lessons encouraged students to keep track of and report on the time they spent watching TV or videos, or playing video games, to motivate them to limit those activities on their own. The initial lessons were followed by TV-Turn off, an organization that encourages less TV viewing. For ten days, students were challenged to go without television, videos, or video games. After that, teachers encouraged the students to stay within a media allowance of seven hours per week. Almost all students participated in the Turnoff, and most stayed under their budget for the following weeks.

Additional lessons encouraged children to use their time more selectively, and many of the final lessons had students themselves advocate reducing screen activities. This study is by no means the first to find a link between television and violence. Virtually all of 3,500 research studies on the subject in the past 40 years have shown the same relationship, according to the American Academy of Pediatrics. Among the most noteworthy studies is Dr. Leonard D. Eron’s, which found that exposure to television violence in childhood is the strongest predictor of aggressive behavior later in life—stronger even than violent behavior as children.

The more violent television the subjects watched at age eight, the more serious was their aggressive behavior even 22 years later. Another study by Dr. Brandon S. Centerwall found that murder rates climb after the introduction of television. In the United States and Canada, murder rates doubled 10 to 15 years after the introduction of television, after the first TV generation grew up. Centerwall tested this pattern in South Africa, where television broadcasts were banned until 1975.

Murder rates in South Africa remained relatively steady from the mid-1940s through the mid-1970s. By 1987, however, the murder rate had increased 130 percent from its 1974 level. The murder rates in the United States and Canada had leveled1 off in the meantime. Centerwall’s study implies that the medium of television, not just the con-tent, promotes violence, and the current study by Dr Robinson supports that conclusion.

The Turnoff did not specifically target violent television, nor did the following allowance period. Reducing television in general reduces aggressive behavior. Even television that is not “violent” is more violent than real life and may lead viewers to believe that violence is funny, inconsequential, and a viable solution to problems. Also, watching television of any content robs us of the time to interact with real people. Watching too much TV may inhibit the skills and patience we need to get along with others without resorting to aggression. TV, as a medium, pro-motes aggression and violence. The best solution is to turn it off.

Questions 14-20. Complete the summary using words from the box below.

A study that was published in January 2001 found that when children (14)… ……………………. less, they behaved less (15) ……………………… Students in a California elementary school participated in the study, which lasted (16) ……………………….. By the end of the study, title children’s behavior had changed. For example, the children’s (17) …………………………. reported that the children were acting less violently than before. During the study, the children kept a record of the (18)…………………….they watched TV. Then, for ten days, they (19) ……………………… Near the end of the study, the students began to suggest watching (20)………………

Questions 21-24

Do the following statements agree with the information in Reading Passage 2? In boxes 21-24 write

TRUE if the statement is true according to the passage.

FALSE if the statement contradicts the passage.

NOT GIVEN if there is no information about this in the passage.

21 Only one study has found a connection between TV and violent behavior.

22 There were more murders in Canada after people began watching TV.

23 The United States has more violence on TV than other countries.

24 TV was introduced in South Africa in the 1940s.

Questions 25 and 26

For each question, choose the correct letter A-D and write it in boxes 25 and 26 on your Answer Sheet.

25 According to the passage,

A only children are affected by violence on TV

B only violent TV programs cause violent behavior

C children who watch too much TV get poor grades in school

D watching a lot of TV may keep us from learning important social skills

26 The authors of this passage believe that

A some violent TV programs are funny

B the best plan is to stop watching TV completely

C it’s better to watch TV with other people than on your own

D seven hours a week of TV watching is acceptable

SECTION 3

Issues Affecting the Southern Resident Orcas

Orcas, also known as killer whales, are opportunistic feeders, which means they will take a variety of different prey species. J, K, and L pods (specific groups of orcas found in the region) are almost exclusively fish eaters. Some studies show that up to 90 percent of their diet is salmon, with Chinook salmon being far and away their favorite. During the last 50 years, hundreds of wild runs of salmon have become extinct due to habitat loss and overfishing of wild stocks. Many of the extinct salmon stocks are the winter runs of Chinook and coho. Although the surviving stocks have probably been sufficient to sustain the resident pods, many of the runs that have been lost were undoubtedly traditional resources favored by the resident orcas. This may be affecting the whales’ nutrition in the winter and may require them to change their patterns of movement in order to search for food. Other studies with tagged whales have shown that they regularly dive up to 800 feet in this area. Researchers tend to think that during these deep dives the whales may be feeding on bottomfish. Bottomfish species in this area would include halibut, rockfish, lingcod, and greenling. Scientists estimate that today’s lingcod population in northern Puget Sound and the Strait of Georgia is only 2 percent of what it was in 1950. The average size of rockfish in the recreational catch has also declined by several inches since the 1970s, which is indicative of overfishing. In some locations, certain rockfish species have disappeared entirely. So even if bottomfish are not a major food resource for the whales, the present low numbers of available fish increases the pressure on orcas and all marine animals to find food. (For more information on bottomfish see the San Juan County Bottomfish Recovery Program.)

Toxic substances accumulate in higher concentrations as they move up the food chain. Because orcas are the top predator in the ocean and are at the top of several different food chains in the environment, they tend to be more affected by pollutants than other sea creatures. Examinations of stranded killer whales have shown some extremely high levels of lead, mercury, and polychlorinated hydrocarbons. Abandoned marine toxic waste dumps and present levels of industrial and human refuse pollution of the inland waters probably presents the most serious threat to the continued existence of this orca population. Unfortunately, the total remedy to this huge problem would be broad societal changes on many fronts. But because of the fact that orcas are so popular, they may be the best species to use as a focal point in bringing about the many changes that need to be made in order to protect the marine environment as a whole from further toxic poisoning.

C The waters around the San Juan Islands are extremely busy due to international commercial shipping, fishing, whale watching, and pleasure boating. On a busy weekend day in the summer, it is not uncommon to see numerous boats in the vicinity of the whales as they travel through the area. The potential impacts from all this vessel traffic with regard to the whales and other marine animals in the area could be tremendous. The surfacing and breathing space of marine birds and mammals is a critical aspect of their habitat, which the animals must consciously deal with on a moment-to-moment basis throughout their lifetimes. With all the boating activity in the vicinity, there are three ways in which surface impacts are most likely to affect marine animals: (a) collision, (b) collision avoidance, and (c) exhaust emissions in breathing pockets. The first two impacts are very obvious and don’t just apply to vessels with motors. Kayakers even present a problem here because they’re so quiet. Marine animals, busy hunting and feeding under the surface of the water, may not be aware that there is a kayak above them and actually hit the bot-tom of it as they surface to breathe. The third impact is one most people don’t even think of. When there are numerous boats in the area, especially idling boats, there are a lot of exhaust fumes being spewed out on the surface of the water. When the whale comes up to take a nice big breath of “fresh” air, it instead gets a nice big breath of exhaust fumes. It’s hard to say how greatly this affects the animals, but think how breathing polluted air affects us (i.e., smog in large cities like Los Angeles, breathing the foul air while sitting in traffic jams, etc.).

D Similar to surface impacts, a primary source of acoustic pollution for this population of orcas would also be derived from the cumulative underwater noise of vessel traffic. For cetaceans, the underwater sound environment is perhaps the most critical component of their sensory and behavioral lives. Orcas communicate with each other over short and long distances with a variety of clicks, chirps, squeaks, and whistles, along with using echolocation to locate prey and to navigate. They may also rely on passive listening as a primary sensory source. The long-term
impacts from noise pollution would not likely show up as noticeable behavioral changes in habitat use, but rather as sensory damage or gradual reduction in population health. A new study at The Whale Museum called the SeaSound Remote Sensing Network has begun studying underwater acoustics and its relationship to orca communication.

Questions 27-30

Reading Passage 3 has four sections (A-D). Choose the most suitable heading for each section from the list of headings below. Write the appropriate numbers (i-vii) in boxes 27-30 on your Answer Sheet. There are more headings than sections, so you will not use all of them.

List of headings
i Top Ocean Predators
ii Toxic Exposure
iii Declining Fish Populations
iv Pleasure Boating in the San Juan Islands
v Underwater Noise
vi Smog in Large Cities
vii Impact of Boat Traffic

27 Section A
28 Section B
29 Section C
30 Section D

Questions 31-32

For each question, choose the appropriate letter A-D and write it in boxes 31 and 32 on your Answer Sheet.

31 Killer whales (orcas) in the J, K, and L pods prefer to eat
A halibut
B a type of salmon
C a variety of animals
D fish living at the bottom of the sea

32 Some groups of salmon have become extinct because
A they have lost places to live
B whales have eaten them
C they don’t get good nutrition
D the winters in the area are too cold

Questions 33-40. Complete the chart below. Write NO MORE THAN THREE WORDS.

View answers – Alternative transportation

How did it go? Please share your feedback in the comment section below:

ALBERT EINSTEIN IELTS Reading

SECTION 1

Albert Einstein is perhaps the best-known scientist of the 20th century. He received the Nobel Prize in Physics in 1921 and his theories of special and general relativity are of great importance to many branches of physics and astronomy. He is well known for his theories about light, matter, gravity, space and time. His most famous idea is that energy and mass are different forms of the same thing.


Einstein was born in Wurttemberg, Germany on 14th March 1879. His family was Jewish but he had not been very religious in his youth although he became very interested in Judaism in later life. It is well documented that Einstein did not begin speaking until after the age of three. In fact, he found speaking so difficult that his family were worried that he would never start to speak. When Einstein was four years old, his father gave him a magnetic compass. It was this compass that inspired him to explore the world of science. He wanted to understand why the needle always pointed north whichever way he turned the compass. It looked as if the needle was moving itself. But the needle was inside a closed case, so no other force (such as the wind) could have been moving it. And this is how Einstein became interested in studying science and mathematics.


In fact, he was so clever that at the age of 12 he taught himself Euclidean geometry. At fifteen, he went to school in Munich which he found very boring. he finished secondary school in Aarau, Switzerland and entered the Swiss Federal Institute of Technology in Zurich from which he graduated in 1900. But Einstein did not like the teaching there either. He often missed classes and used the time to study physics on his own or to play the violin instead. However, he was able to pass his examinations by studying the notes of a classmate. His teachers did not have a good opinion of him and refused to recommend him for a university position. So, he got a job in a patent office in Switzerland. While he was working there, he wrote the papers that first made him famous as a great scientist.


Einstein had two severely disabled children with his first wife, Mileva. His daughter (whose name we do not know) was born about a year before their marriage in January 1902. She was looked after by her Serbian grandparents until she died at the age of two. It is generally believed that she died from scarlet fever but there are those who believe that she may have suffered from a disorder known as Down Syndrome. But there is not enough evidence to know for sure. In fact, no one even knew that she had existed until Einstein’s granddaughter found 54 love letters that Einstein and Mileva had written to each other between 1897 and 1903. She found these letters inside a shoe box in their attic in California. Einstein and Mileva’s son, Eduard, was diagnosed with schizophrenia. He spent decades in hospitals and died in Zurich in 1965. Just before the start of World War I, Einstein moved back to Germany and became director of a school there. But in 1933, following death threats from the Nazis, he moved to the United States, where he died on 18th April 1955.


Questions 1-8
Do the following statements agree with the information given in the text? For questions 1-8, write:

TRUE if the statement agrees with the information

FALSE if the statement contradicts the information

NOT GIVEN if there is no information on this 

1. The general theory of relativity is a very important theory in modern physics.

2. Einstein had such difficulty with language that those around him thought he would never learn how to speak.

3. It seemed to Einstein that nothing could be pushing the needle of the compass around except the wind.

4. Einstein enjoyed the teaching methods in Switzerland.

5. Einstein taught himself how to play the violin.

6. His daughter died of schizophrenia when she was two.

7. The existence of a daughter only became known to the world between 1897 and 1903.

8. In 1933 Einstein moved to the United States where he became an American citizen. 

Questions 9-10 

Complete the sentences below. Choose NO MORE THAN THREE WORDS from the text for each answer. 

He tried hard to understand how the needle could seem to move itself so that it always (9)………………. 

He often did not go to classes and used the time to study physics (10)…………………..or to play music. 

Questions 11-13 

Choose the correct letter, A, B, C or D

11. The name of Einstein’s daughter

A was not chosen by him.

B is a mystery.

C is shared by his granddaughter.

D was discovered in a shoe box. 

12. His teachers would not recommend him for a university position because

A they did not think highly of him.

B they thought he was a Nazi.

C his wife was Serbian.

D he seldom skipped classes. 

13. The famous physicist Albert Einstein was of

A Swiss origin.

B Jewish origin.

C American origin.

D Austrian origin. 

SECTION 2

DRINKING FILTERED WATER 

A The body is made up mainly of water. This means that the quality of water that we drink every day has an important effect on our health. Filtered water is healthier than tap water and some bottled water. This is because it is free of contaminants, that is, of substances that make it dirty or harmful. Substances that settle on the bottom of a glass of tap water and microorganisms that carry diseases (known as bacteria or germs) are examples of contaminants. Filtered water is also free of poisonous metals and chemicals that are common in tap water and even in some bottled water brands. 

B The authorities know that normal tap water is full of contaminants and they use chemicals, such as chlorine and bromine in order to disinfect it. But such chemicals are hardly safe. Indeed, their use in water is associated with many different conditions and they are particularly dangerous for children and pregnant women. For example, consuming bromine for a long time may result in low blood pressure, which may then bring about poisoning of the brain, heart, kidneys and liver. Filtered water is typically free of such water disinfectant chemicals. 

C Filtered water is also free of metals, such as mercury and lead. Mercury has ended up in our drinking water mainly because the dental mixtures used by dentists have not been disposed of safely for a long time. Scientists believe there is a connection between mercury in the water and many allergies and cancers as well as disorders, such as ADD, OCD, autism and depression. 

D Lead, on the other hand, typically finds its way to our drinking water due to pipe leaks. Of course, modern pipes are not made of lead but pipes in old houses usually are. Lead is a well-known carcinogen and is associated with pregnancy problems and birth defects. This is another reason why children and pregnant women must drink filtered water. 

E The benefits of water are well known. We all know, for example, that it helps to detoxify the body, So, the purer the water we drink, the easier it is for the body to rid itself of toxins. The result of drinking filtered water is that the body does not have to use as much of its energy on detoxification as it would when drinking unfiltered water. This means that drinking filtered water is good for our health in general. That is because the body can perform all of its functions much more easily and this results in improved metabolism, better weight management, improved joint lubrication as well as efficient skin hydration. 

F There are many different ways to filter water and each type of filter targets different contaminants. For example, activated carbon water filters are very good at taking chlorine out. Ozone water filters, on the other hand, are particularly effective at removing germs. 

G For this reason, it is very important to know exactly what is in the water that we drink so that we can decide what type of water filter to use. A Consumer Confidence Report (CCR) should be useful for this purpose. This is a certificate that is issued by public water suppliers every year, listing the contaminants present in the water. If you know what these contaminants are, then it is easier to decide which type of water filter to get. 

Questions 14-20 The text has seven paragraphs, A-G. 

Which paragraph contains the following information? 

14. a short summary of the main points of the text

15. a variety of methods used for water filtration

16. making it easier for the body to get rid of dangerous chemicals

17. finding out which contaminants your water filter should target

18. allergies caused by dangerous metals 

19. a dangerous metal found in the plumbing of old buildings

20. chemicals of cleaning products that destroy bacteria 

Questions 21-26 Do the following statements agree with the information given in the text? For questions 21-26, write: 

TRUE if the statement agrees with the information

FALSE if the statement contradicts the information

NOT GIVEN if there is no information on this 

21. The type of water you consume on a regular basis has a great impact on your overall health and wellness.

22. Filtered water typically contains water disinfectant chemicals.

23. Exposure to disinfectant chemicals is linked with poisoning of the vital organs.

24. Drinking tap water helps minimise your exposure to harmful elements.

25. People wearing artificial teeth are more likely to be contaminated.
26. People who are depressed often suffer from dehydration. 

SECTION 3

SPEECH DYSFLUENCY AND POPULAR FILLERS 

A speech dysfluency is any of various breaks, irregularities or sound-filled pauses that we make when we are speaking, which are commonly known as fillers. These include words and sentences that are not finished, repeated phrases or syllables, instances of speakers correcting their own mistakes as they speak and “words” such as ‘huh’, ‘uh’, ‘erm’, ‘urn’, ‘hmm’, ‘err’, ‘like’, ‘you know’ and ‘well’. 

Fillers are parts of speech which are not generally recognised as meaningful and they include speech problems, such as stuttering (repeating the first consonant of some words). Fillers are normally avoided on television and films, but they occur quite regularly in everyday conversation, sometimes making up more than 20% of “words” in speech. But they can also be used as a pause for thought. 

Research in linguistics has shown that fillers change across cultures and that even the different English speaking nations use different fillers. For example, Americans use pauses such as ‘um’ or ’em’ whereas the British say ‘uh’ or ‘eh’. Spanish speakers say ‘ehhh’ and in Latin America (where they also speak Spanish) but not Spain, ‘este’ is used (normally meaning ‘this’). 

Recent linguistic research has suggested that the use of ‘uh’ and ‘um’ in English is connected to the speaker’s mental and emotional state. For example, while pausing to say ‘uh’ or ‘um’ the brain may be planning the use of future words. According to the University of Pennsylvania linguist Mark Liberman, ‘um’ generally comes before a longer or more important pause than ‘uh’. At least that’s what he used to think. 

Liberman has discovered that as Americans get older, they use ‘uh’ more than ‘um’ and that men use ‘uh’ more than women no matter their age. But the opposite is true of ‘um’. The young say ‘um’ more often than the old. And women say ‘um’ more often than men at every age. This was an unexpected result because scientists used to think that fillers had to do more with the amount of time a speaker pauses for, rather than with who the speaker is. 

Liberman mentioned his finding to fellow linguists in the Netherlands and this encouraged the group to look for a pattern outside American English. They studied British and Scottish English, German, Danish, Dutch and Norwegian and found that women and younger people said ‘um’ more than ‘uh’ in those languages as well. 

Their conclusion is that it is simply a case of language change in progress and that women and younger people are leading the change. And there is nothing strange about this. Women and young people normally are the typical pioneers of most language change. What is strange, however, is that ‘um’ is replacing ‘uh’ across at least two continents and five Germanic languages. Now this really is a mystery. 

The University of Edinburgh sociolinguist Josef Fruehwald may have an answer. In his view, ‘um’ and ‘uh’ are pretty much equivalent. The fact that young people and women prefer it is not significant. This often happens in language when there are two options. People start using one more often until the other is no longer an option. It’s just one of those things. 

As to how such a trend might have gone from one language to another, there is a simple explanation, according to Fruehwald. English is probably influencing the other languages. We all know that in many countries languages are constantly borrowing words and expressions of English into their own language so why not borrow fillers, too? Of course, we don’t know for a fact whether that’s actually what’s happening with ‘um’ but it is a likely story. 

Questions 27-34 Do the following statements agree with the information given in the text? For questions 27-34, write 

TRUE if the statement agrees with the information

FALSE if the statement contradicts the information

NOT GIVEN if there is no information on this 

27. Fillers are usually expressed as pauses and probably have no linguistic meaning although they may have a purpose.

28. In general, fillers vary across cultures.

29. Fillers are uncommon in everyday language.

30. American men use ‘uh’ more than American women do.

31. Younger Spaniards say ‘ehhh’ more often than older Spaniards.

32. In the past linguists did not think that fillers are about the amount of time a speaker hesitates.

33. During a coffee break Liberman was chatting with a small group of researchers.

34. Fruehwald does not believe that there are age and gender differences related to ‘um’ and ‘uh’. 

Questions 35-40. Choose the correct letter, A, B, C or D. 

35. Fillers are not

A used to give the speaker time to think.

B phrases that are restated.

C used across cultures.

D popular with the media. 

36. It had originally seemed to Mark Liberman that 

A ‘um’ was followed by a less significant pause than ‘uh’.

B ‘uh’ was followed by a shorter pause than ‘um’.

C ‘uh’ was followed by a longer pause than ‘um’.

D the use of ‘um’ meant the speaker was sensitive. 

37. Contrary to what linguists used to think, it is now believed that the choice of filler

A may have led to disagreements.

B depends on the characteristics of the speaker.

C has nothing to do with sex.

D only matters to older people. 

38. According to Liberman, it’s still a puzzle why

A a specific language change is so widely spread.

B the two fillers are comparable.

C we have two options.

D ‘um’ is preferred by women and young people. 

39. Concerning the normal changes that all languages go through as time goes by,

A old men are impossible to teach.

B men in general are very conservative.

C young men simply copy the speech of young women.

D women play a more important role than men. 

40. According to Fruehwald, the fact that ‘um’ is used more than ‘uh’

A proves that ‘um’ is less important.

B shows that young people have low standards.

C shows that they have different meanings.

D is just a coincidence. 

View answers – Albert Einstein

How did it go? Please share your feedback in the comment section below:


Show answers
Albert Einstein

1.         True

2.         True

3.         False

4.         False

5.         Not given

6.         Not given

7.         False

8.         Not given

9.         Pointed north

10.       On his own

11.       B

12.       A

13.       B

14.       A

15.       F

16.       E

17.       G

18.       C

19.       D

20.       B

21.       True

22.       False

23.       True

24.       False

25.       Not given

26.       Not given

27.       True

28.       True

29.       False

30.       True

31.       Not given

32.       False

33.       Not given

34.       False

35.       D

36.       B

37.       B

38.       A

39.       D

40.       D

ADHD IELTS Reading


You should spend about 20 minutes on Questions 1-13, which are based on Reading Passage 1 below.

Disorders: An Overview
Autistic Spectrum Disorder

Children with Autistic Spectrum Disorder have difficulty understanding what other people are saying, need help to play with other children, enjoy routines and find unfamiliar situations difficult. People with Autistic Spectrum Disorder can be good at creative activities like art, music and poetry. They can concentrate on one thing for a long time no they can become very good at something that they like doing.
ADHD – Attention Deficit
Hyperactivity Disorder

People with ADHD have three types of problems. Overactive behaviour (hyperactivity), impulsive behaviour and difficulty paying attention. Children with ADHD are not just very active but have a wide range of problem behaviours which can make them very difficult to care for and control. Those who have ADHD often find it difficult to fit in at school. They may also have problems getting on with other children. Some children have significant problems with concentration and attention but are not necessarily overactive or impulsive. These children are sometimes described as having Attention Deficit Disorder (ADD) rather than ADHD. ADD can easily be missed because the child is quiet and dreamy rather than disruptive. ADHD is not related to intelligence. Children with all levels of ability can have ADHD.
Stress
Stress can be defined as the way you feel when you’re under abnormal pressure. All sorts of situations can cause stress. The most common, however. involve work, money matters and relationships with partners, children or other family members. Stress may be caused either by major upheavals and life events such as divorce, unemployment, moving house and bereavement, or by a series of minor irritations such as feeling undervalued at work or dealing with difficult children.
Some stress can be positive and research has suggested that a moderate level of stress makes us perform better. It also makes us more alert and can help us in challenging situations such as job interviews or public speaking. Stressful situations can also be exhilarating and some people actually thrive on the excitement that comes with dangerous sports or other ‘high-risk’ activities.
Schizophrenia
Schizophrenia is a diagnosis given to some people who have severely disrupted beliefs and experiences. During an episode of schizophrenia, a person’s experience and interpretation of the outside world is disrupted – they may lose touch with reality, see or hear things that are not there and act in unusual ways in response to these ‘hallucinations’. An episode of schizophrenia can last for several weeks and can be very frightening. The causes are unknown but episodes of schizophrenia appear to be associated with changes in some brain chemicals. Stressful experiences and some recreational drugs tire sometimes thought to trigger an episode.
Depression
Depression describes a range of moods, from the low spirits that we all experience, to a severe problem that interferes with everyday life. The latter type sometimes referred to as “clinical depression”, is defined as its “a persistent exaggeration of the everyday feelings that accompany sadness”. If you have severe depression you may experience low mood, loss of interest and pleasure as well as feelings of worthlessness and guilt. You may also experience tearfulness, poor concentration, reduced energy, reduced or increased appetite, changes in weight, sleep problems and anxiety. You may even feel that life is not worth living and plan or attempt suicide.
Obsessive-Compulsive Disorder in Adults
Imagine you are getting up in the morning. You know you will need to go to the bathroom, but the thought of accidentally touching the doorknob is frightening. There may be dangerous bacteria on it. Of course, you cleaned the entire bathroom yesterday, including the usual series of spraying disinfectant, washing and rinsing. As usual, it took a couple of hours to do it the right way. Even then you weren’t sure whether you had missed an area, so you had to re-wash the floor. Naturally, the doorknob was sprayed and rubbed three times with a bactericidal spray. Now the thought that you could have missed a spot on the doorknob makes you very nervous.

This description might give you some sense of the tormented and anxious world that people with Obsessive-Compulsive Disorder (OCD) live in. It is a world filled with dangers from outside and from within. Often elaborate rituals and thoughts are used to ward off feared events, but no amount of mental or physical activity seems adequate, so doubt and anxiety are often present.

People who do not have OCD may perform behaviours in a ritualistic way, repeating, checking, or washing things out of habit or concern. Generally, this is done without much, if any, worry. What distinguishes OCD as a psychiatric disorder is that the experience of obsessions, and the performance of rituals, reaches such intensity or frequency that it causes significant psychological distress and interferes in a significant way with psycho-social functioning. The guideline of at least one hour spent on symptoms per day is often used as a measure of ‘significant interference’. However, among patients who try to avoid situations that bring on anxiety and compulsions, the actual symptoms may not consume an hour. Yet the or situations would dearly constitute interfering with functioning. Consider, for instance, a welfare mother who throws out more than $100 of groceries a week because of contamination fears. Although this behaviour has a major effect on her functioning, it might not consume one hour per day.

Patients with OCD describe the experience as having thoughts (obsessions) that they associate with some danger. The sufferer generally recognises that it is his or her own thoughts, rather than something imposed by someone else (as in some paranoid schizophrenic patients). However, the disturbing thoughts cannot be dismissed, and simply nag at the sufferer. Something must then be done to relieve the danger and mitigate the fear. This leads to actions and thoughts that are intended to neutralise the danger. These are the compulsions. Because these behaviours seem to give the otherwise ‘helplessly anxious’ person something to combat the danger, they are temporarily reassuring. However, since the ‘danger’ is typically irrational or imaginary, it simply returns, thereby triggering another cycle of the briefly reassuring compulsions. From the standpoint of classic conditioning, this pattern of painful obsession followed by temporarily reassuring compulsion eventually produces an intensely ingrained habit. It is rare to see obsessions without compulsions.

The two most common obsessions are fears of contamination and fear of harming oneself or others, while the two most common compulsions are checking and cleaning.
Questions 1-5
Look at the statements (Questions I – 5) and the list of disorders (A – G) below. Match each statement with the correct disorder A – G.
NB There are more disorders than descriptions, so you will not use them all.

  1. can be positive in small doses but is generally associated with pressure
  2. feeling that there is danger constantly present
  3. has experiences that may or may not be part of the ‘real’ world
  4. active to the point of losing concentration and becoming disruptive
  5. good at art but not at communicating

    Types of Disorders

A. Stress
B. Autistic Spectrum Disorder
C. Attention Deficit Disorder
D. Schizophrenia
E. Attention Deficit Hyperactivity Disorder
F. Depression
G. Obsessive-Compulsive Disorder

Questions 6-9

Complete the table below. Write NO MORE THAN THREE WORDS from the passage for each answer.

Questions 10-13 

Choose the correct Utter, A, B, C or D. 

10.Which disorder could cause visible physical changes?

A.Autistic Spectrum Disorder

B.Stress

C.Schizophrenia

D.Depression

11.Episodes of which disorder may last for a limited period of time?

A.ADHD

B.Autistic Spectrum Disorder

C.Schizophrenia

D.Depression

12.Which disorder can be triggered by the death of a loved one?

A.Autistic Spectrum Disorder

B.ADHD

C.Stress

D.OCD

13.What characterises sufferers of OCD?

A.the fear of going outside

B.the performance of rituals 

C. the desire to hurt others 

D. the feeling that they are helpless to ease their distress 

Reading Passage 2 

You should spend about 20 minutes on Questions 14 – 26, which are based on Reading Passage 2 below. 

The Developing World 

A.THE DEVELOPING WORLD – the economically underdeveloped countries of Asia. Africa. Oceania and Latin America – is considered as an entity with common characteristics, such as poverty, high birth rates, and economic dependence on the advanced countries. Until recently, the developing world was known as ‘the third world’. The French demographer Alfred Sauvy coined the expression (in French) in 1952 by analogy with the ‘third estate’ – the commoners of France before and during the French Revolution – as opposed to priests and nobles, comprising the First and second estates respectively. ‘Like the third estate’, wrote Sauvy, ‘the third world is nothing, and it wants to be something’. The term, therefore, implies that the third world is exploited, much as the third estate was exploited and that, like the third estate, its destiny is a revolutionary one. It conveys as well a second idea, also discussed by Sauvy – that of nonalignment, for the developing world belongs neither to the industrialised capitalist world nor to the industrialised former communist bloc. The expression ‘third world’ was used at the 1955 conference of Afro-Asian countries held in Bandung. Indonesia. In 1956 a group of social scientists associated with Sauvy’s National Institute of Demographic Studies, in Paris, published a book called ‘Le Tiers-Monde’. Three years later, the French economist Francois Perroux launched a new journal, on problems of underdevelopment, with the same title. By the end of the 1950s, the term was frequently employed in the French media to refer to the underdeveloped countries of Asia. Africa, Oceania and Latin America. Present-day politicians and social commentators, however, now use the term ‘developing world’ in a politically correct effort to dispel the negative connotations of ‘third world’. 

B. Countries in the developing world have a number of common traits: distorted and highly dependent economies devoted to producing primary products for the developed world; traditional, rural social structures; high population growth and widespread poverty. Nevertheless, the developing world is sharply differentiated, for it includes countries on various levels of economic development. And despite the poverty of the countryside and the urban shantytowns, the ruling elites of most third world countries are wealthy. 

C. This combination of conditions in Asia, Africa, Oceania and Latin America is linked to the absorption of the developing world into the international capitalist economy, by way of conquest or indirect domination. The main economic consequence of Western domination was the creation, for the first time in history, of a world market. By setting up sub-economies linked to the West throughout the developing world, and by introducing other modern institutions, industrial capitalism disrupted traditional economies and, indeed, societies. This disruption led to underdevelopment. 

D. Because the economies of underdeveloped countries have been geared to the needs of industrialised countries, they often comprise only a few modem economic activities, such as mining or the cultivation of plantation crops. Control over these activities has often remained in the hands of large foreign firms. The prices of developing world products are usually determined by large buyers in the economically dominant countries of the West, and trade with the West provides almost all the developing world’s income. Throughout the colonial period, outright exploitation severely limited the accumulation of capital within the foreign-dominated countries. Even after decolonisation (in the 1950s, 1960s, and 1970s), the economies of the developing world grew slowly, or not at all, owing largely to the deterioration of the ‘terms of trade’ – the relationship between the cost of the goods a nation must import from abroad and its income from the exports it sends to foreign countries. Terms of trade are said to deteriorate when the cost of imports rises faster than income from exports. Since buyers in the industrialised countries determined the prices of most products involved in international trade, the worsening position of the developing world was scarcely surprising. Only the oil-producing countries – after 1973 – succeeded in escaping the effects of Western domination of the world economy. 

E. No study of the developing world could hope to assess its future prospects without taking into account population growth. While the mortality rate from poverty-related diseases continues to cause international concern, the birth rate continues to rise at unprecedented levels. This population explosion in the developing world will surely prevent any substantial improvements in living standards, as well as threaten people in stagnant economies with worsening poverty and starvation levels. 

Questions 14-18 Reading Passage 2 has five paragraphs, A – E. 

Write the appropriate number i – viii in spaces 14-18 below. 

Choose the most suitable heading for each paragraph from the list of headings below. 

List of Headings 

i. The great divide between rich and poor. 

ii. The status and destiny of the developing’ world follow a European precedent. 

iii. Economic progress in the developing world slowed down In political unrest. 

iv. More people, less food. 

v. Western countries refuse to acknowledge their history of colonisation. 

vi. Open trade is the main reason these countries become impoverished. 

vii. Rivalry in the developing world between capitalist and former communist countries. 

vii. Prices and conditions set by outsiders 

14. paragraph A_______________ 

15. Paragraph B_______________ 

16. Paragraph C_______________ 

17. Paragraph D_______________ 

18. Paragraph E_______________ 

Questions 19-22 

Do the following statements agree with the information given in Reading Passage 2? 

19. Agriculture still plays a role in the economy of developing countries. _______ 

20.The population of the developing world increases at such a fast ________ rate because they constantly need to renew the labour force. ________ 

21.Countries that spend more on imports than came from exports can experience problems. ________

22.As the developing world, oil-rich countries are also victims of dominance by Western powers.________

Questions 23-26

Write the correct letter A – F in spaces 23 – 26 below. 

23.Countries in the developing world

24.The term ‘the third world’ implies

25.One factor that is prevalent in the developing world is

26.One consequence of the terms of trade was

A.economic dependence on developed countries.

B.that decolonisation took a long time to achieve.

C.dictate the needs of industrialised countries.

D.share common characteristics.

E.that many economies stagnated.

F.a society that wants something it does not have.

Reading Passage 3

You should spend about 20 minutes on Questions 27-40, which are based on Reading Passage 3 below. 

Biometrics 

A.The term “biometrics’ is derived from the Greek words bio (life) and metric (to measure). It refers totechnologies for measuring and analysing a person’s physiological or behavioural characteristics, such asfingerprints, irises, voice patterns, facial patterns and hand measurements, for identification andverification purposes. One of the earliest known examples of biometrics in practice was a form offingerprinting used in China in the 14th century. Chinese merchants stamped children’s palm prints andfootprints on paper with ink to distinguish the young children from one another. This method ofbiometrics is still being practiced today.

B.Until the late 1800s, identification largely relied upon ‘photographic memory.’ In the 1890s, an anthropologist and police desk clerk in Paris named Alphonse Bertillon sought to fix the problem of identifying convicted criminals and turned biometrics into a distinct field of study. He developed amethod of multiple body measurements which was named after him – Bertillonage. Bertillon based his system on the claim that the measurement of adult bones does not change after the age of 20. He also introduced a cataloguing system, which enabled the filing and checking of records quite quickly. His system was used by police authorities throughout the world, until 1903, when two identical measurements were obtained for two different persons at Fort Leavenworth prison. The prisons witched to fingerprinting the following day and the rest of the world soon followed. abandoning Bertillonage forever. After the failure of Bertillonage, the police started using fingerprinting, which was developed by Richard Edward Henry of Scotland Yard, essentially reverting to the same methods used by the Chinese for years.

C.In the past three decades, biometrics has moved from a single method (fingerprinting) to more thanten different methods. Hundreds of companies are involved with this development and continue toimprove their methods as the technology available to them advances. As the industry grows, however,so does the public concern over privacy issues. Laws and regulations continue to be drafted andstandards are beginning to be developed. While no other biometric has yet reached a wide range of useof fingerprinting, some are beginning to be used in both legal and business areas.

D.Identification and verification have long been in practice by presenting a personal document, such asa license, ID card or a passport. It may also require personal information such as passwords or PINs. Forsecurity reasons, often two, or all three, of these systems are combined but as times progress, we are inconstant need for more secure and accurate measures. Authentication by biometric verification isbecoming increasingly common in corporate and public security systems, consumer electronics andpoint-of-sale applications. In addition to security, the driving force behind biometric verification hasbeen convenience. Already, many European countries are introducing a biometric passport which willcarry a paper-thin computer chip to store the facial image and at least one additional biometricidentifier. This will help to counter fraudulent efforts to obtain duplicate passports and will verify theidentity of the holder against the document.

E.Identification and verification are mainly used today in the fight against crime with the methods offingerprint and DNA analysis. It is also used in security for granting access rights by voice patternrecognition. Additionally, it is used for personal comfort by identifying a person and changing personalsettings accordingly, as in setting car seats by facial recognition. Starting in early 2000, the use ofbiometrics in schools has become widespread, particularly in the UK and USA. A number of justificationsare given for such practices, including combatting truancy and replacing library cards or meal cards withfingerprinting systems. Opponents of school biometrics have raised privacy concerns against thecreation of databases that would progressively include the entire population.

F.Biometric devices consist of a reader or scanning device, software that converts the gatheredinformation into digital form, and a database that stores the biometric data for comparison withprevious records. When converting the biometric input, the software identifies specific points of data asmatch points. The match points are processed using an algorithm into a value that can be comparedwith biometric data in the database. There are two types of biometrics: behavioural and physical.Behavioural biometrics are generally used for verification while physical biometrics can be used foreither identification or verification.

G.Iris-pattern and retina-pattern authentication methods are already employed in some bank automatic teller machines. Voice waveform recognition, a method of verification that has been used for many years with tape recordings in telephone wiretaps, is now being used for access to proprietary data banks in research facilities. Facial-recognition technology has been used by law enforcement to pick out individuals in large crowds with considerable reliability. Hand geometry is being used in the industry to provide physical access to buildings. Earlobe geometry has been used to disprove the identity of individuals who claim to be someone they are not (identity theft). Signature comparison is not as reliable, all by itself, like other biometric verification methods but offers an extra layer of verification when used in conjunction with one or more other methods. No matter what biometric methodology is used, the identification verification process remains the same. A record of a person’s unique characteristic is captured and kept in a database. Later on, when identification verification is required, a new record is captured and compared with the previous record in the database. If the data in the new record matches that in the database record, the person’s identity is confirmed. 

H. As technology advances and time goes on, more and more private companies and public utilities will use biometrics for safe, accurate identification. However, these advances will raise many concerns throughout society, where many may not be educated on the methods. Some believe this technology can cause physical harm to an individual using it, or that instruments used are unsanitary. For example, there are concerns that retina scanners might not always be clean. There are also concerns as to whether our personal information taken through biometric methods can be misused, tampered with, or sold, eg. by criminals stealing, rearranging or copying the biometric data Also, the data obtained using biometrics can be used in unauthorized ways without the individual’s consent. Much still remains to be seen in the effectiveness of biometric verification before we can identify it as the safest system for identification. 

Questions 27-31 

Reading Passage 3 has eight paragraphs, A-H. 

Write the correct letter A – H in spaces 27 – 31 below. 27. possible health hazards associated with the use of biometrics 

28. convicted criminals were not the first to be identified by the use of biometrics 

29. the application of mathematics in assessing biometric data 

30. despite its limitations, biometrics has become a commercial field of activity 

31. some biometric methods are useful only in conjunction with others 

Questions 32-34 

Complete the sentences below. 

Choose NO MORE THAN TWO WORDS from the passage for each answer. 

32. Members of the public are becoming increasingly worried about the……………….that may accompany the use of biometrics. 

33. Biometrics can be used to improve the……………….of drivers and passengers. 

34. Regardless of the technology used, it has one common purpose: to find somebody’s ………………and store it on the computer. 

Questions 35-40 

Write the correct letter A-L in spaces 35-40 below. 

Biometrics 

As long ago as the 14th century, the Chinese made use of biometrics in order to tell young children apart, but it was only in the 1890s when it was first used by the authorities as a means of 35 ………………………… in criminal cases. The system developed by the Frenchman Bertillon – that of measuring adult bones – was flawed, however, and so police adopted 36 ………………………… as a more reliable way of identifying suspects. Governments, companies and even schools employ biometric technology to ensure, for example, that people do not enter a country illegally, gain access to certain buildings, or assume someone else’s 37 …………………………. Apart from security, another important 38 behind biometric verification has been 39 …………………………. The use of biometrics, however, has its critics, who say that the data collected could be used for different purposes without our 40 ………………

A. identification
B security 
C convenience 
D scanning 
E fingerprinting 
F identity 
G violation 
H measuring 
I justification 
J approval 
K factor 
L apprehension 

How did it go? Please share your feedback in the comment section below:

View answers – ADHD IELTS Reading

Adam’s Wine IETLS reading test

 SECTION 1

 A Water is the giver and, at the same time, the taker of life. It covers most of the surface of the planet we live on and features large in the development of the human race. On present predictions, it is an element that is set to assume even greater significance.

B Throughout history, water has had a huge impact on our lives. Humankind has always had a rather ambiguous relationship with water, on the one hand receiving enormous benefit from it, not just as a drinking source, but as a provider of food and a means whereby to travel and to trade. But forced to live close to water in order to survive and to develop, the relationship has not always been peaceful or beneficial. In fact, it has been quite the contrary. What has essentially been a necessity for survival has turned out in many instances to have a very destructive and life-threatening side.

C Through the ages, great floods alternated with long periods of drought have assaulted people and their environment, hampering their fragile fight for survival. The dramatic changes to the environment that are now a feature of our daily news are not exactly new: fields that were once lush and fertile are now barren; lakes and rivers that were once teeming with life are now long gone; savannah has been turned to desert. What perhaps is new is our naive wonder when faced with the forces of nature.

D Today, we are more aware of climatic changes around the world. Floods in far-flung places are instant news for the whole world. Perhaps these events make us feel better as we face the destruction of our own property by floods and other natural disasters.

E In 2002, many parts of Europe suffered severe flood damage running into billions of euros. Properties across the continent collapsed into the sea as waves pounded the coastline wreaking havoc with sea defences. But it was not just the seas. Rivers swollen by heavy rains and by the effects of deforestation carried large volumes of water that wrecked many communities.

F Building stronger and more sophisticated river defences against flooding is the expensive short-term answer. There are simpler ways. Planting trees in highland areas, not just in Europe but in places like the Himalayas, to protect people living in low-lying regions like the Ganges Delta, is a cheaper and more attractive solution. Progress is already being made in convincing countries that the emission of carbon dioxide and other greenhouse gases is causing considerable damage to the environment. But more effort is needed in this direction. 

G And the future? If we are to believe the forecasts, it is predicted that two-thirds of the world population will be without fresh water by 2025. But for a growing number of regions of the world the future is already with us. While some areas are devastated by flooding, scarcity of water in many other places is causing conflict. The state of Texas in the United States of America is suffering a shortage of water with the Rio Grande failing to reach the Gulf of Mexico for the first time in 50 years in the spring of 2002, pitting region against region as they vie for water sources. With many parts of the globe running dry through drought and increased water consumption, there is now talk of water being the new oil. 

H Other doom-laden estimates suggest that, while tropical areas will become drier and uninhabitable, coastal regions and some low-lying islands will in all probability be submerged by the sea as the polar ice caps melt. Popular exotic destinations now visited by countless tourists will become no-go areas. Today’s holiday hotspots of southern Europe and elsewhere will literally become hotspots – too hot to live in or visit. With the current erratic behaviour of the weather, it is difficult not to subscribe to such despair. 

I Some might say that this despondency is ill-founded, but we have had ample proof that there is something not quite right with the climate. Many parts of the world have experienced devastating flooding. As the seasons revolve, the focus of the destruction moves from one continent to another. The impact on the environment is alarming and the cost to life depressing. It is a picture to which we will need to become accustomed. 

Questions 1-8. Reading Passage 1 has eight paragraphs labelled A-I. Choose the most suitable headings for paragraphs B-I from the list of headings below. 

Write the appropriate numbers (i-xiii) in boxes 1-8 on your answer sheet. One of the headings has been done for you as an example. Note: There are more headings than paragraphs, so you will not use all of them. 

Example: Paragraph A — Answer vii 

List of Headings

i Environmental change has always been with us

ii The scarcity of water

iii Rivers and seas cause damage

iv Should we be despondent? Or realistic?

v Disasters caused by the climate make us feel better

vi Water, the provider of food

vii What is water?

viii How to solve flooding

ix Far-flung flooding

x Humans’ relationship with water

xi The destructive force of water in former times

xii Flooding in the future

xiii A pessimistic view of the future 

1 Paragraph B

2 Paragraph C

3 Paragraph D

4 Paragraph E

5 Paragraph F

6 Paragraph G

7 Paragraph H

8 Paragraph I 

Questions 9-15 Choose the appropriate letters A-D and write them in boxes 9-15 on your answer sheet. 

9 The writer believes that water

A is gradually becoming of greater importance.

B will have little impact on our lives in future.

C is something we will need more than anything else.

D will have even greater importance in our lives in the future. 

10 Humankind’s relationship with water has been

A two-sided.

B one-sided.

C purely one of great benefit.

D fairly frightening. 

11 The writer suggests that

A we are in awe of the news we read and see on TV every day.

B change to the environment leaves us speechless.

C we should not be in awe of the news we read and see on TV every day.

D our surprise at the environmental change brought about by nature is something new. 

12 According to the text, planting trees

A has to be coordinated internationally.

B is more expensive than building sea and river defences.

C is a less expensive answer to flooding than building river defences.

D is not an answer to the problem of flooding in all regions. 

13 By 2025, it is projected that

A at least half the world population will have fresh water.

B the majority of the world population will have fresh water. 

C one-third of the world population will have fresh water.

D fresh water will only be available to half of the world population. 

14 According to the text, in the future low-lying islands

A will still be habitable.

B will not be under water.

C are likely to be under water.

D will probably not be under water. 

15 According to the writer,

A people do not need to get used to environmental damage.

B people will need to get used to climate changes that cause environmental damage.

C people are now more used to environmental damage than they have been in the past.

D the general despondency about environmental changes is ill-founded. 

Section 2 

Is it any wonder that there are teacher shortages? Daily, the press carries reports of schools going on four-day weeks simply because they cannot recruit enough teachers. But why? There is no straightforward answer. For a start, fewer students are entering teacher-training courses when they leave school. But can you blame young people after the barracking faced by the teaching profession in the UK over the last decade? The attack, relentless in the extreme, has been on several fronts. Government inspectors, by accident or design, have been feeding the media a constant stream of negative information about the teaching establishments in this country. Teachers also come in for a lot of flak from politicians. And the government wonders why there are problems in schools. 

The government’s obvious contempt for the teaching profession was recently revealed by one of the most powerful people in government when she referred to schools as ‘bog standard comprehensives’. Hardly the sort of comment to inspire parents or careers advisers seeking to direct young people’s future. Would you want to spend your working life in a dead-end profession? The government doesn’t seem to want you to either. 

On the administrative side, most teachers are weighed down by an increasing flow of bureaucracy. Cynicism would have me believe that this stops teachers from fomenting dissent as they are worn out by useless administrative exercises. Most teachers must then also be cynics! 

Teacher bashing has, unfortunately, spread to youngsters in schools as the recent catalogue of physical attacks on teachers will testify. If grown-ups have no respect for the teaching profession, young people can hardly be expected to think any differently. The circle is then squared when, as well as experienced, competent teachers being driven out of the profession by the increased pressure and stress; fewer students are applying for teacher-training courses. 

Increased salaries are certainly welcome, but they are not the complete answer to a sector in crisis. Addressing the standing of the profession in the eyes of the public is crucial to encourage experienced teachers to remain in the classroom and to make it an attractive career option for potential teachers once again. It might also be a good idea for the relevant ministers to go on a fact-finding mission and find out from teachers in schools, rather than relying overmuch on advisers, as to what changes could be brought about to improve the quality of the education service. Initiatives in the educational field surprisingly come from either politicians who know little about classroom practice or educational theorists who know even less, but are more dangerous because they work in the rarefied air of universities largely ignorant of classroom practice. 

Making sure that nobody without recent classroom experience is employed as a teacher-trainer at any tertiary institution would further enhance the teaching profession. If someone does not have practical experience in the classroom, they cannot in all seriousness propound theories about it. Instead of being given sabbaticals to write books or papers, lecturers in teacher-training establishments should be made to spend a year at the blackboard or, these days, the whiteboard. This would give them practical insights into current classroom practice. Student teachers could then be given the chance to come and watch the specialists in the classroom: a much more worthwhile experience than the latter sitting thinking up ideas far removed from the classroom. Then we would have fewer initiatives like the recent government proposal to teach thinking in school. Prima facie, this is a laudable recommendation. But, as any practising teacher will tell you, this is done in every class. Perhaps someone needs to point out to the academic who thought up the scheme that the wheel has been around for some time. 

In the educational field, there is surprisingly constant tension between the educational theorists and government officials on the one hand, who would like to see teachers marching in unison to some greater Utopian abstraction and, on the other, practising teachers. Any experienced classroom practitioner knows that the series of initiatives on teaching and learning that successive governments have tried to foist on schools and colleges do not work. 

Questions 16-22 Complete the summary below of the first four paragraphs of Reading Passage 2. Choose ONE WORD ONLY from the passage for each answer. Write your answers in boxes 16-22 on your answer sheet. 

Is it surprising that there is a 16______ of teachers? Schools do not have enough teachers, but what are the reasons for this? To begin with, fewer students are going into 17______ after finishing school. But this is not young people’s fault. The 18 ______ of teaching has been under constant attack over the last ten years. The government’s lack of respect for the profession is 19______ . Moreover, administratively, the flow of bureaucracy is 20______. Even pupils in schools have no respect for those who teach them, as a 21______ series of assaults on teachers shows. The growing strain and stress means that, as well as fewer applications for teacher-training courses, teachers who have experience and are 22______ are also being driven out. 

Questions 23-29 In boxes 23-29 on your answer sheet, write 

YES if the statement agrees with the claims of the writer

NO if the statement contradicts the claims of the writer

NOT GIVEN if it is impossible to say what the writer thinks about this 

23 More students are entering teacher-training courses.

24 The government is right to be surprised that there are problems in schools.

25 Teachers are too weighed down by administrative duties to stir up trouble.

26 All teachers are cynics.

27 Politicians are not as dangerous as educational theorists, who know even less than the former about educational theory.

28 Any experienced classroom practitioner knows that the initiatives on teaching and learning that governments have tried to impose on schools do not work.

29 The government’s attitude with regard to teachers is of great interest to the general public. 

Question 30 Choose the appropriate letter A-D and write it in box 30 on your answer sheet. 

30 Which one of the following is the most suitable title for the passage?

A Politicians and teachers.

B A profession undervalued.

C Recruitment difficulties in the teaching profession.

D Teacher-training needs improvement. 

Section 3 

In one corner of the room is a mass of tangled rope suspended from the ceiling with some sections dangling to the floor; the first of three encountered pieces of work that have a resounding impact on the viewing public. 

It stops one in one’s tracks: how dare it be there – this mess of nothing! It is like arranged chaos: that is, the confused mixture of varying sizes of rope, dipped in latex, looks as though it might collapse in a heap on the floor at any moment. At the same time, it is held up and in place by a series of fine wires and hooks, giving it a strange sense of… order. 

A deliberate challenge to the forces of gravity. It is a shambles. It makes one laugh. It is play. It is drawing in the air! Maybe it can move or dance about! Yet, it is hardly there, like something imagined. 

The materials are cheap and disposable. Impermanent, like … the people looking at it. But it is very definitely present! It has a presence. You can see that people want to walk into it and become a part of it – but alas! The gallery guard is hovering nearby. 

To the left of this piece, running along the wall, in two rows on top of each other, is a long series of lid-less boxes. They are mounted at average nose height and are made of fibreglass which gives them a shiny, almost moist, appearance. They are the colour of murky water, absorbing the gallery light with an opacity similar to that of mucus or tree gum. 

They look as though they might be soft and malleable to touch, with their irregular edges and non-conforming sides. This gives the overall impression that they could fall in on themselves or slide down the wall. The structure is puzzlingly familiar, similar to things in the world, and yet it is not like anything in particular. 

In the adjacent corner is the third piece, consisting of a collection of nine cylindrical open-ended objects, slit part way from end to end. They give the appearance of being randomly placed – some lying, some leaning on the wall or on each other-all seeming somehow to be related. Like the boxes, they are a multiple of each other. Made of fibreglass with a shiny surface they look almost like abandoned pods that had once been alive. The associations seem to jump around in one’s head, running between sensations of delight and pleasure, violence and discomfort. 

One has to bend down to be with them more. Driven by the desire to physically interact, one is almost forced to stoop further so that one can touch, or indeed taste, this intriguing surface; but no, the guard is there. 

The visual language apparent in these artworks is unfamiliar, as is the artist, Eva Hesse. Her work is as exciting as it is disturbing. For many, Hesse’s sculpture refers essentially to the body. This, perhaps, does not seem surprising when it is in relation to the body that women are generally assessed. Hesse died of a brain tumour in 1970 at the age of 34. It must be an inescapable inevitability, therefore, that her work was read in the context of its time where it has, until recently, been largely abandoned. 

Given the influence of feminism on our cultural consciousness since that period, it seems paramount that we avoid, or at the very least attempt to avoid, those dramatic facts about her life and family history. We may then be freed from a limited and narrow translation of her art. 

Hesse’s work is much more ambiguous and funny than some rather literal readings would have us believe. Perhaps it is precisely because her use of metaphor in her work is so subtle that it escapes the one-line definitions we so love to employ. 

We are now, more than ever, hungry for the cult of ‘personality’. While Hesse and others before and since can more than fill that demand, we seem in danger of focusing on the life of the artist and not on the life of the art. 

When looking at Hesse’s sculpture, drawings and paintings, the most interesting and challenging aspects lie just there – within the work. And this must be the starting point for any interpretation, not her complex life or untimely death. 

Questions 31-36 In boxes 31-36 on your answer sheet, write 

YES if the statement agrees with the claims of the writer

NO if the statement contradicts the claims of the writer

NOT GIVEN if it is impossible to say what the writer thinks about this Example answer: The Guggenheim Art Gallery is in New York. Yes 

31 The first piece of Hesse’s art has little effect on visitors to the gallery. 32 The order inherent in the first piece of Hesse’s art is essential to the understanding of her work.

33 The second piece of art by Hesse is inferior in several significant ways to the first.

34 The second piece by Hesse has several design faults that attract the public.

35 The third piece of work arouses different emotions.

36 Of the three pieces of Hesse’s work described, the first is the writer’s favourite. 

Question 37-40 Choose the appropriate letter A-D and write it in box 30 on your answer sheet. 

37 According to the writer, Eva Hesse

A is not a well-known artist.

B is very familiar, as is her work.

C is not a good artist.

D is strongly attracted by visual language. 

38 The writer concludes that

A Hesse’s work is timeless.

B the understanding of Hesse’s work has until recently been interpreted only in the context of its time.

C Hesse’s work is a product of her time and is not relevant to the modern world.

D Hesse’s work is easy to read. 

39 The writer thinks that it is ……………… to define Hesse’s work.

A not difficult B essential

C not important.

D not easy 

40 In the present climate,

A we may lose sight of Hesse’s art and focus on her life.

B personality is very important.

C art cults are in vogue.

D we may lose sight of Hesse’s life and focus on her art. 

How did it go? Please share your feedback in the comment section below:

View answers Adam’s Wine 

 A Remarkable Beetle IETLS reading

 SECTION 1

Some of the most remarkable beetles are the dung beetles, which spend almost their whole lives eating and breeding in dung’. 

 More than 4,000 species of these remarkable creatures have evolved and adapted to the world’s different climates and the dung of its many animals. Australia’s native dung beetles are scrub and woodland dwellers, specialising in coarse marsupial droppings and avoiding the soft cattle dung in which bush flies and buffalo flies breed.

In the early 1960s George Bornemissza, then a scientist at the Australian Government’s premier research organisation, the Commonwealth Scientific and Industrial Research Organisation (CSIRO), suggested that dung beetles should be introduced to Australia to control dung-breeding flies. Between 1968 and 1982, the CSIRO imported insects from about 50 different species of dung beetle, from Asia, Europe and Africa, aiming to match them to different climatic zones in Australia. Of the 26 species that are known to have become successfully integrated into the local environment, only one, an African species released in northern Australia, has reached its natural boundary.

Introducing dung beetles into a pasture is a simple process: approximately 1,500 beetles are released, a handful at a time, into fresh cow pats in the cow pasture. The beetles immediately disappear beneath the pats digging and tunnelling and, if they successfully adapt to their new environment, soon become a permanent, self-sustaining part of the local ecology. In time they multiply and within three or four years the benefits to the pasture are obvious.

Dung beetles work from the inside of the pat so they are sheltered from predators such as birds and foxes. Most species burrow into the soil and bury dung in tunnels directly underneath the pats, which are hollowed out from within. Some large species originating from France excavate tunnels to a depth of approximately 30 cm below the dung pat. These beetles make sausage-shaped brood chambers along the tunnels. The shallowest tunnels belong to a much smaller Spanish species that buries dung in chambers that hang like fruit from the branches of a pear tree. South African beetles dig narrow tunnels of approximately 20 cm below the surface of the pat. Some surface-dwelling beetles, including a South African species, cut perfectly-shaped balls from the pat, which are rolled away and attached to the bases of plants.

For maximum dung burial in spring, summer and autumn, farmers require a variety of species with overlapping periods of activity. In the cooler environments of the state of Victoria, the large French species (2.5 cms long) is matched with smaller (half this size), temperate-climate Spanish species. The former are slow to recover from the winter cold and produce only one or two generations of offspring from late spring until autumn. The latter, which multiply rapidly in early spring, produce two to five generations annually. The South African ball-rolling species, being a subtropical beetle, prefers the climate of northern and coastal New South Wales where it commonly works with the South African tunnelling species. In warmer climates, many species are active for longer periods of the year.

Dung beetles were initially introduced in the late 1960s with a view to controlling buffalo flies by removing the dung within a day or two and so preventing flies from breeding. However, other benefits have become evident. Once the beetle larvae have finished pupation, the residue is a first-rate source of fertiliser. The tunnels abandoned by the beetles provide excellent aeration and water channels for root systems. In addition, when the new generation of beetles has left the nest the abandoned burrows are an attractive habitat for soil-enriching earthworms. The digested dung in these burrows is an excellent food supply for the earthworms, which decompose it further to provide essential soil nutrients. If it were not for the dung beetle, chemical fertiliser and dung would be washed by rain into streams and rivers before it could be absorbed into the hard earth, polluting water courses and causing blooms of blue-green algae. Without the beetles to dispose of the dung, cow pats would litter pastures making grass inedible to cattle and depriving the soil of sunlight. Australia’s 30 million cattle each produce 10-12 cow pats a day. This amounts to 1.7 billion tonnes a year, enough to smother about 110,000 sq km of pasture, half the area of Victoria. 

Dung beetles have become an integral part of the successful management of dairy farms in Australia over the past few decades. A number of species are available from the CSIRO or through a small number of private breeders, most of whom were entomologists with the CSIRO’s dung beetle unit who have taken their specialised knowledge of the insect and opened small businesses in direct competition with their former employer. 

Questions 1-5

Do the following statements reflect the claims of the writer in Reading Passage 1? In boxes 1-5 on your answer sheet write

YES if the statement reflects the claims of the writer

NO if the statement contradicts the claims of the writer

NOT GIVEN if it is impossible to say what the writer thinks about this

  1. Bush flies are easier to control than buffalo flies.
  2. Four thousand species of dung beetle were initially brought to Australia by the CSIRO.
  3. Dung beetles were brought to Australia by the CSIRO over a fourteen-year period.
  4. At least twenty-six of the introduced species have become established in Australia.
  5. The dung beetles cause an immediate improvement to the quality of a cow pasture. Questions 6-8

Label the tunnels on the diagram below. Choose your labels from the box given with the diagram Reading

Reading Passage 2


Section A

The role of governments in environmental management is difficult but inescapable. Sometimes, the state tries to manage the resources it owns, and does so badly. Often, however, governments act in an even more harmful way. They actually subsidise the exploitation and consumption of natural resources. A whole range of policies, from farm- price support to protection for coal-mining, do environmental damage and (often) make no economic sense. Scrapping them offers a two-fold bonus: a cleaner environment and a more efficient economy. Growth and environmentalism can actually go hand in hand, if politicians have the courage to confront the vested interest that subsidies create.

Section B

No activity affects more of the earth’s surface than farming. It shapes a third of the planet’s land area, not counting Antarctica, and the proportion is rising. World food output per head has risen by 4 per cent between the 1970s and 1980s mainly as a result of increases in yields from land already in cultivation, but also because more land has been brought under the plough. Higher yields have been achieved by increased irrigation, better crop breeding, and a doubling in the use of pesticides and chemical fertilisers in the 1970s and 1980s.

Section C

All these activities may have damaging environmental impacts. For example, land clearing for agriculture is the largest single cause of deforestation; chemical fertilisers and pesticides may contaminate water supplies; more intensive farming and the abandonment of fallow periods tend to exacerbate soil erosion; and the spread of mono-Culture and use of high-yielding varieties of crops have been accompanied by the disappearance of old varieties of food plants which might have provided some insurance against pests or diseases in future. Soil erosion threatens the productivity of land in both rich and poor countries. The United States, where the most careful measurements have been done, discovered in 1982 that about one-fifth of its farmland as losing topsoil at a rate likely to diminish the soil’s productivity. The country subsequently embarked upon a program to convert 11 per cent of its cropped land to meadow or forest. Topsoil in India and China is vanishing much faster than in America.

Section D

Government policies have frequently compounded the environmental damage that farming can cause. In the rich countries, subsidies for growing crops and price supports for farm output drive up the price of land. The annual value of these subsidies is immense: about $250 billion, or more than all World Bank lending in the 1980s.To increase the output of crops per acre, a farmer’s easiest option is to use more of the most readily available inputs: fertilisers and pesticides. Fertiliser use doubled in Denmark in the period 1960-1985 and increased in The Netherlands by 150 per cent. The quantity of pesticides applied has risen too; by 69 per cent in 1975-1984 in Denmark, for example, with a rise of 115 per cent in the frequency of application in the three years from 1981. In the late 1980s and early 1990s some efforts were made to reduce farm subsidies. The most dramatic example was that of New Zealand, which scrapped most farm support in 1984. A study of the environmental effects, conducted in 1993, found that the end of fertiliser subsidies had been followed by a fall in fertiliser use (a fall compounded by the decline in world commodity prices, which cut farm incomes). The removal of subsidies also stopped land-clearing and over-stocking, which in the past had been the principal causes of erosion. Farms began to diversify. The one kind of subsidy whose removal appeared to have been bad for the environment was the subsidy to manage soil erosion. In less enlightened countries, and in the European Union, the trend has been to reduce rather than eliminate subsidies, and to introduce new payments to encourage farmers to treat their land in environmentally friendlier ways, or to leave it follow. It may sound strange but such payments need to be higher than the existing incentives for farmers to grow food crops. Farmers, however, dislike being paid to do nothing. In several countries they have become interested in the possibility of using fuel produced from crop residues either as a replacement for petrol (as ethanol) or as fuel for power stations (as biomass). Such fuels produce far less carbon dioxide than coal or oil, and absorb carbon dioxide as they grow. They are therefore less likely to contribute to the greenhouse effect. But they die rarely competitive with fossil fuels unless subsidised – and growing them does no less environmental harm than other crops.

Section E

In poor countries, governments aggravate other sorts of damage. Subsidies for pesticides and artificial fertilisers encourage farmers to use greater quantities than are needed to get the highest economic crop yield. A study by the International Rice Research Institute Of pesticide use by farmers in South East Asia found that, with pestresistant varieties of rice, even moderate applications of pesticide frequently cost farmers more than they saved. Such waste puts farmers on a chemical treadmill: bugs and weeds become resistant to poisons, so next year’s poisons must be more lethal. One cost is to human health, every year some 10,000 people die from pesticide poisoning, almost all of them in the developing countries, and another 400,000 become seriously ill. As for artificial fertilisers, their use world-wide increased by 40 per cent per unit of farmed land between the mid 1970s and late 1980s, mostly in the developing countries. Overuse of fertilisers may cause farmers to stop rotating crops or leaving their land fallow. That, In turn, may make soil erosion worse.

Section F

A result of the Uruguay Round of world trade negotiations is likely to be a reduction of 36 per cent in the average levels of farm subsidies paid by the rich countries in 1986-1990. Some of the world’s food production will move from Western Europe to regions where subsidies are lower or non-existent, such as the former communist countries and parts of the developing world. Some environmentalists worry about this outcome. It will undoubtedly mean more pressure to convert natural habitat into farmland. But it will also have many desirable environmental effects. The intensity of farming in the rich world should decline, and the use of chemical inputs will diminish. Crops are more likely to be grown up in the environments to which they are naturally suited. And more farmers in poor countries will have the money and the incentive to manage their land in ways that are sustainable in the long run. That is important. To feed an increasingly hungry world, farmers need every incentive to use their soil and water effectively and efficiently.

Questions 14-18

Reading Passage 2 has six sections A-F. Choose the most suitable headings for sections A-D and F from the list of headings below. Write the appropriate numbers i-ix in boxes 14-18 on your answer sheet.

List of Headings

i The probable effects of the new international trade agreement

ii The environmental impact of modern farming

iii Farming and soil erosion

iv The effects of government policy in rich countries

v Governments and management of the environment

vi The effects of government policy in poor countries

vii Farming and food output

viii The effects of government policy on food output

ix The new prospects for world trade

14 Paragraph A

15 Paragraph B

16 Paragraph C

17 Paragraph D

18 Paragraph F

Questions 19-22

Complete the table below using the information in sections B and C of Reading Passage 2. Choose your answers AG from the box below the table and write them in boxes 19-22 on your answer sheet.

Questions 23-27

Choose the appropriate letters A-D and write them in boxes 23-27 on your answer sheet.

23 Research completed in 1982 found that in the United States soil erosion

A reduced the productivity of farmland by 20 per cent.

B was almost as severe as in India and China.

C was causing significant damage to 20 per cent of farmland.

D could be reduced by converting cultivated land to meadow or forest.

24 By the mid-1980s, farmers in Denmark

A used 50 per cent less fertiliser than Dutch farmers.

B used twice as much fertiliser as they had in 1960.

C applied fertiliser much more frequently than in 1960.

D more than doubled the amount of pesticide they used in just 3 years.

25 Which one of the following increased in New Zealand after 1984?

A farm incomes

B use of fertilizer

C over-stocking

D farm diversification

26 The writer refers to some rich countries as being ‘less enlightened’ than New Zealand because

A they disapprove of paying farmers for not cultivating the land.

B their new fuel crops are as harmful as the ones they have replaced.

C their policies do not recognise the long-term benefit of ending subsidies.

D they have not encouraged their farmers to follow environmentally friendly practices.

27 The writer believes that the Uruguay Round agreements on trade will

A encourage more sustainable farming practices in the long term.

B do more harm than good to the international environment.

C increase pressure to cultivate land in the rich countries.

D be more beneficial to rich than to poor countries.

Question 28

From the list below choose the most suitable title for Reading Passage 2. Write the appropriate letter A-E in box 28 on your answer sheet.

A Environmental management

B Increasing the world’s food supply

C Soil erosion

D Fertilisers and pesticides – the way forward

E Farm subsidies

The Concept of Role Theory

Role set

Any individual in any situation occupies a role in relation to other people. The particular individual with whom one is concerned in the analysis of any situation is usually given the name of focal person. He has the focal role and can be regarded as sitting in the middle of a group of people, with whom he interacts in some way in that situation. This group of people is called his role set. For instance, in the family situation, an individual’s role set might be shown as in Figure 6. The role set should include all those with whom the individual has more than trivial interactions.

Role definition

The definition of any individual’s role in any situation will be a combination of the role expectations that the members of the role set have of the focal role. These expectations are often occupationally denned, sometimes even legally so. The role definitions of lawyers and doctors are fairly clearly defined both in legal and in cultural terms. The role definitions of, say, a film star or bank manager, are also fairly clearly defined in cultural terms, too clearly perhaps.

Individuals often find it hard to escape from the role that cultural traditions have defined for them. Not only with doctors or lawyers is the required role behaviour so constrained that if you are in that role for long it eventually becomes part of you, part of your personality. Hence, there is some likelihood that all accountants will be alike or that all blondes are similar – they are forced that way by the expectations of their role.

It is often important that you make it clear what your particular role is at a given time. The means of doing this are called, rather obviously, role signs. The simplest of role signs is a uniform. The number of stripes on your arm or pips on your shoulder is a very precise role definition which allows you to do certain very prescribed things in certain situations. Imagine yourself questioning a stranger on a dark street at midnight without wearing the role signs of a policeman!

In social circumstances, dress has often been used as a role sign to indicate the nature and degree of formality of any gathering and occasionally the social status of people present. The current trend towards blurring these role signs in dress is probably democratic, but it also makes some people very insecure. Without role signs, who is to know who has what role?

Place is another role sign. Managers often behave very differently outside the office and in it, even to the same person. They use a change of location to indicate a change in role from, say, boss to friend. Indeed, if you wish to change your roles you must find some outward sign that you are doing so or you won’t be permitted to change – the subordinate will continue to hear you as his boss no matter how hard you try to be his friend. In very significant cases of role change, e.g. from a soldier in the ranks to officer, from bachelor to married man, the change of role has to have a very obvious sign, hence rituals. It is interesting to observe, for instance, some decline in the emphasis given to marriage rituals. This could be taken as an indication that there is no longer such a big change in role from single to married person, and therefore no need for a public change in sign.

In organisations, office signs and furniture are often used as role signs. These and other perquisites of status are often frowned upon, but they may serve a purpose as a kind of uniform in a democratic society; roles without signs often lead to confused or differing expectations of the role of the focal person.

Role ambiguity

Role ambiguity results when there is some uncertainty in the minds, either of the focal person or of the members of his role set, as to precisely what his role is at any given time. One of the crucial expectations that shape the role definition is that of the individual, the focal person himself. If his occupation of the role is unclear, or if it differs from that of the others in the role set, there will be a degree of role ambiguity. Is this bad? Not necessarily, for the ability to shape one’s own role is one of the freedoms that many people desire, but the ambiguity may lead to role stress which will be discussed later on. The virtue of job descriptions is that they lessen this role ambiguity.

Unfortunately, job descriptions are seldom complete role definitions, except at the lower end of the scale. At middle and higher management levels, they are often a list of formal jobs and duties that say little about the more subtle and informal expectations of the role. The result is therefore to give the individual an uncomfortable feeling that there are things left unsaid, i. e. to heighten the sense of role ambiguity.

Looking at role ambiguity from the other side, from the point of view of the members of the role set, lack of clarity in the role of the focal person can cause insecurity, lack of confidence, irritation and even anger among members of his role set. One list of the roles of a manager identified the following: executive, planner, policy maker, expert, controller of rewards and punishments, counsellor, friend, teacher. If it is not clear, through role signs of one sort or another, which role is currently the operational one, the other party may not react in the appropriate way — we may, in fact, hear quite another message if the focal person speaks to us, for example, as a teacher and we hear her as an executive.

Questions 29-35

Do the following statements reflect the views of the writer in Reading Passage 3? In boxes 29-35 on your answer sheet write

YES if the statement reflects the views of the writer

NO if the statement contradicts the views of the writer

NOT GIVEN if it is impossible to know what the writer thinks about this

29 It would be a good idea to specify the role definitions of soldiers more clearly.

30 Accountants may be similar to one another because they have the same type of job.

31 It is probably a good idea to keep dress as a role sign even nowadays.

32 The decline in emphasis on marriage rituals should be reversed.

33 Today furniture operates as a role sign in the same way as dress has always done.

34 It is a good idea to remove role ambiguity.

35 Job descriptions eliminate role ambiguity for managers.

Questions 36-39

Choose ONE OR TWO WORDS from Reading Passage 3 for each answer. Write your answers in boxes 36-39 on your answer sheet.

36 A new headmaster of a school who enlarges his office and puts in expensive carpeting is using the office as a ………………….

37 The graduation ceremony in many universities is an important……………….

38 The wig which judges wear in UK courts is a………………….

39 The parents of students in a school are part of the headmaster’s………………………

Question 40

Choose the appropriate letter A-D and write it in box 40 on your answer sheet. This text is taken from

A a guide for new managers in a company.

B a textbook analysis of behaviour in organisations.

C a critical study of the importance of role signs in modern society.

D a newspaper article about role changes.

How did it go? Please share your feedback in the comment section below:

View answers: A Remarkable Beetle